You are on page 1of 86

Test 2 Sample Essay - Score of 6

Though America has suffered through times of massive economic distress where government intervention was necessary in maintaining the peoples spirit, America was founded by individuals at town meetings, behind their ploughs, or in peaceful petition. In todays relatively stable society, people should feel empowered to make changes and solve problems because countless reforms began at the grassroots level. If people need inspiration or proof, they need only to look to the progressives or the Steimetz twins, who rid the beaches of cigarette butts with their activism. In the early 1900s, American government had been congealed into a state of corruption and inaction by the powerful lobbyists of the industries. In this Gilded Age, the disparity between rich and poor were astonishingwhile Vanderbilt became a millionaire, millions of factory workersmostly immigrants lived in squalor and ate insalubrious food. At this same time, the progressive movement sought to rectify these injustices. Jane Addams, for example, started a settlement house for women workers, where she not only provided child care and financial support, but also campaigned for the abolition of child labor and the creation of laws to protect workers safety. The actions of hundreds of others like her eventuly led to reforms in city governmentthe referendum, the city council ran by experts, not lobbyists, etc. All these reforms originated at the movement of the individual. However, movement by individuals does not have to spread nationwide to be effective. In the state of Connecticut, two teenagers who passionately cared for the environment effected state laws to make littering of cigarette butts illegal on beaches. The teens began with simple cleanup campaigns with friends and family but realized that more action couldand shouldbe taken. Thence, they began to make trips to state legislature in addition to trips to the beachinstead of collecting butts, they rallied support. Eventually, a law was passed to make their sentiments permanenthowever, the impetus started in the minds of two teenagers. Government is powerful and well-intentioned, but the sordid mechanics of politics often obstruct problem solving. Individuals, however, are not so hampered. In a country where rights not enumerated (explicitly given to the federal govt) are for the people, people should see individual action as a right AND responsibility. Score Explanation Demonstrating outstanding critical thinking, this essay effectively and insightfully develops a point of view (In a country where rights not enumerated . . . are for the people, people should see individual action as a right AND responsibility ), using clearly appropriate examples from history and current events to support the position. Well organized and clearly focused as it discusses people who were empowered to make changes and solve problems, the response displays clear coherence and smooth progression of ideas (In the state of Connecticut, two teenagers who passionately cared for the environment effected state laws to make littering of cigarette butts illegal on beaches. . . . Eventually, a law was passed to make their sentiments permanent ). The essay also exhibits skillful use of language, using a varied, accurate, and apt vocabulary, and meaningful variety in sentence structure is evident ( Government is powerful and wellintentioned, but the sordid mechanics of politics often obstruct problem solving ). This essay demonstrates clear and consistent mastery and receives a score of 6. Sample

Essay - Score of 5
Many Americans believe that the government should become more involved with the community and solve all of the problems. Although this government was created to promote the general welfare of the nation, it was not created to become involved in every aspect of peoples lives. This nation was created to give everyone the same oppertunities not solve all the problems of the american people. The more the government becomes involved w/ the everyday lives of americans and the nations economy the less free this society will become. The more power that is given to the government the more the government will exert its influence on the american people. The american people should not give up freedoms for safety. Like Benjamin Franklin once said [someone] who is willing to give up essential freedoms for safety does not deserve freedom or safety. The government of the United States was founded on the belief of the individuals power to shape his future. The government is here not to simply help the individual, it is here to make sure that the individual can help themselves. The

government should not be as active as it is in american society. As long as someone can help themselves the government should not be involved. When creating this nation, the founding fathers strove to limit the power of the national government. But, over the last two hundred years the freedom of the individual has shrunk while the power of the government has grown. For this nation to be considered free and for this economy to be considered a free economy the government has to re-ignite their belief in the american worker. The government has to believe that americans can take care of themselves. The more the government becomes involved in this society, the more this society will become slow, unproductive, and freedomless. The Federal government has become more and more involved in this society because it has lost its faith in the american worker. The liberals of the national government try to help those that dont need help and unintentionaly limit the freedoms of the people. This nation needs to re-kindle its faith and believe that people can help themselves and believe that capitalism and competition will take care of the problems in this society. Every time this nation doubts the founding principles of this country another man loses his freedoms. The country that was founded in order to limit government has now instead started to limit individual freedoms under the guise of safety. Government involvement doesnt work, just go to the DPS office. Score Explanation Demonstrating strong critical thinking, this essay effectively develops a point of view ( This nation was created to give everyone the same oppertunities not solve all the problems of the american people ) and provides appropriate reasons and examples to support the position. The essay is focused and well organized as it argues against government influence, displaying coherence and progression of ideas (The government is here not to simply help the individual, it is here to make sure that the individual can help themselves. The government should not be as active as it is in american society ). Using appropriate vocabulary, the response exhibits facility in the use of language, and variety in sentence structure is evident (Every time this nation doubts the founding principles of this country another man loses his freedoms). To earn a score of 6, the writer should use critical thinking to provide additional detailed evidence that supports his or her claim that government involvement leads to a society that will become slow, unproductive, and freedomless. Despite some minor errors, this essay demonstrates reasonably consistent mastery and merits a score of 5.

Sample Essay - Score of 5


Have you ever wondered what it would be like to live in a country where all of your choices are made for you by the government? Your job, your marriage, your future, and voice are freedoms that, unlike the United States, are solely held in the hands of the government. A major problem with this kind of system is the loss of any oppurtunity to solve community or nation-wide issues and problems. Individuals have no right to act independently or self-sufficiently. Althugh the government is essential for solving problems such as creating jobs, providing a military, funding schools, etcetera, they cannot possibly solve all of the worlds problems. That is when the people of America take advantage of our freedom and step up to solve our problems. In contrast to common assumptons, one person can make an enormous difference in the world. For example, every year the famous talk show host, Oprah Winfrey, takes a trip to Africa. Here, she greets the poor children and donates school supplies, shoes, and various items to provide a learning environment. She has even gone as far as building a school and providing teachers with a salary for the next five years. This establishment is a feat that will ensure success and progression in a once helpless village. Oprah is a woman who uses her ability and resources to change lives of so many people worldwide. In addition, UNICEF is an organization that collects money to aid the poverty stricken, ill, and alone children in foreign countries. Millions of dollars are donated to this organization to medicate and save lives of children, but the money is not given by the government; it is given by individuals. Locally speaking, our shcool raised about 2500 dollars by having individual students carry around a small, orange donation box. Every coin collected was really worth something. With only one dollar, a child could recieve vaccination for measles for four years. Lastly, with the recent catastrophe in Asia, the tsunami, thusands of people died, leaving even more people alone and helpless. Our government could only give a minimal amount of money to aid within our budget, so many individuals have stepped up to help. For example, celebrities such as Kobe Bryant and Britney Spears have promised to donate thousands of dollars with each point scored or concert ticket sold. These are also examples of how people can be responsible in changing the world.

In conclusion, we can all make a difference is the problem solving that needs to be done around us. Our government does the best they can, but their ability is limited. Part of living in a democracy is taking advantage of the right to speak and act freely. We need to take pride in this and heal the world with all of our ability. Score Explanation This essay demonstrates strong critical thinking by effectively developing a point of view (Althugh the government is essential for solving problems such as creating jobs, providing a military, funding schools, etcetera, they cannot possibly solve all of the worlds problems) and providing generally appropriate examples of people and organizations who make a difference as support. Well organized and focused as it moves through its examples, the essay displays coherence and progression of ideas (For example, every year the famous talk show host, Oprah Winfrey, takes a trip to Africa. . . . She has even gone as far as building a school and providing teachers with a salary for the next five years. This establishment is a feat that will ensure success and progression in a once helpless village). Through the use of appropriate vocabulary, the response exhibits facility in the use of language, and variety in sentence structure appears throughout (Our government could only give a minimal amount of money to aid within our budget, so many individuals have stepped up to help ). In order to achieve a score of 6, the writer needs to develop the specific examples more fully. Also, the essay contains some minor errors and awkward phrases that need to be revised. Displaying reasonably consistent mastery, this essay earns a score of 5. Sample Essay - Score

of 4

Vote. Vote. Vote. Voting is all one hears about during an election year, and prehaps society should listen. The right to vote is the right to make changes in society. With this right men and women of the United States can do virtually anything. However, most Americans feel that one vote doesnt count. This frame of mind results in a lot of non-voting Americans. And if one doesnt vote then one can not change society. People should take more responsibility for solving problems that affect their communities and their nation. The people who live nearby or those whom live on the other side of the nation ultimately decide how their country functions. Whether society believes it or not, the choices we make by voting, helping around the community, and how we interact within our society affects us directly. By voting Americans get the privledge to decide who runs the nation. This is strictly a privledge as many countries dont have the option to choose who makes their decisions. Americans should take responsibility for their actions and vote on who they feel is the best canidate for the presidency. In doing so, the choices the American people want or need to occur may happen indirectly from their voting. Throughout ones community there are several places, things, or people who could use help. The road is broken, the library is run down, the school needs new books. All of these things can be handled by citizens of a community. Instead of complaining about how the government should fix the problems, the people who live in that community should take responsibility and help. The people that live in a community need to be supportive of each other as well. Helping the homeless, educating low income families, or getting bad influences off of the streets will better a community without the governments involvement. Overall, I feel that the people of a society should take more responsibility for their actions and problems. Governmental support is helpful; however, dependency on the government to make decisions one may not agree upon is not. American people should strive to become more independent in their lives and decision making. Score Explanation This essay demonstrates competent critical thinking in developing its point of view ( People should take more responsibility for solving problems that affect their communities and their nation ), supporting the position with adequate reasons and examples. Generally organized and focused as it discusses the importance of voting and solving community problems, the essay exhibits some coherence and progression of ideas (By voting Americans get the privledge to decide who runs the nation. . . . Americans should take responsibility for their actions and vote on who they feel is the best canidate for the presidency. In doing so, the choices the American people want or need to occur may happen indirectly from their voting). The essay uses generally appropriate vocabulary and some variety in sentence structure appears(Instead of complaining about how the government should fix the problems, the people who live in that community should take responsibility and help ). In order to achieve a higher

score, the writer needs to use critical thinking to provide additional specific evidence to support the position. Despite some errors, the response displays adequate mastery and earns a score of 4.

Sample Essay - Score of 4


People should not rely on the government alone to make their decisions for them. The foundation of the United States is based on radical opinions and was set up by individuals. If people become to lazy to solve their own problems what else will they lose? People could not only become too lazy to make decisions in government or communities, they will most likely become too lazy to decide on family matters or simple everyday problems. If this occurs it will eventually lead to the downfall of our nation. People always complain about the government doing the wrong things or making the wrong decision, but what they dont realize is that they can do something about it. The right for people to make the decion and have individual voices heard has always been important in America. People have the right to vote for whoever they want for president and can put anyone they want into other government offices. People can protest for whatever cause they feel is needed. PETA holds a very strong opinion against the harm of animals while others feel the need to stop the possession of guns. In the seventies many people had their voices heard at Vietnam War protests. It was broadcast even more after the Kent State shooting. The government might not always make the right decisions like at the Kent State shooting the Ohio National Gaurd chose to fire on students. No one wanted this to happen and it only caused further protest. Martin Luther King, Jr., didnt feel the oppresion of black people was fair, the government was wrong. He made a choice to stand-up for his rights and eventually prevailed. If people today start abandoning their right to choose the nation may very well end up like Germany or Cuba. They let someone else make their decisions for them and quickly lost every right they ever possessed. Hopefully, the American people will not let their freedom so easily slip away and will continue to let their own opinions and decisions shine. Decisions can be simple or life-altering, but no matter how tough or impertinent they are people shouldnt hand them over to someone else for them to make. If people dont start using their voice the country will be nothing more than a dictatorship with unintelligent, uninformed zombies as its population. Dont let your voice be drowned out by others. Score Explanation This essay demonstrates competent critical thinking, developing its point of view ( People should not rely on the government alone to make their decisions for them ) with adequate reasoning and examples from recent history. The essay is generally organized and focused as it describes people who make, or made, their voices heard in order to bring about change, displaying some coherence and progression of ideas ( Martin Luther King, Jr., didnt feel the oppresion of black people was fair, the government was wrong. He made a choice to stand-up for his rights and eventually prevailed ). Despite some errors in grammar, usage, and mechanics, the essay uses vocabulary that is generally appropriate and exhibits some variety in sentence structure ( Hopefully, the American people will not let their freedom so easily slip away and will continue to let their own opinions and decisions shine). In order to achieve a higher score, the writer needs to exhibit more control in his or her use of language and use critical thinking to develop each point more fully instead of jumping from example to example. Demonstrating adequate mastery, this essay receives a score of 4. Section #4: View Explanations

1 Explanation for Correct Answer C. Choice (C) is correct. Predictable means capable of being predicted in advance. To foresee something is to know it beforehand. The structure of the sentence indicates that the part of the sentence after the colon supports or further explains the idea in the first part of the sentence. A predictable movie is one in which viewers can foresee plot developments, or determine what will happen in advance, so the terms logically complete the sentence.

Choice (A) is incorrect. Convincing means having power to convince one of the truth, rightness, or reality of something. To misinterpret something is to understand it wrongly. The structure of the sentence indicates that the part of the sentence after the colon supports or further explains the idea in the first part of the sentence. Although one might describe a movies plot as having the ability to convince a viewer of the truth of something, a viewers misinterpretation of the fate of the villain is not evidence of the movies convincing plot. Additionally, there is no reason to believe that knowing what befell the hero would cause a viewer to misunderstand what would eventually happen to the villain. Choice (B) is incorrect. Misleading means giving a wrong impression or leading astray. To anticipate something is to expect it or look forward to it as certain. The structure of the sentence indicates that the part of the sentence after the colon supports or further explains the idea in the first part of the sentence. Although a movie might contain scenes meant to mislead the audience, it does not really make sense to describe an entire plot as misleading; the sentence does not indicate in what way the whole movie leads viewers astray. Additionally, if the plot leads the viewers astray, it is unlikely that they would be able to anticipate, or expect, a certain outcome for the movies villain. Choice (D) is incorrect. In this context, ironic means containing incongruity between the actual result and an expected result. To endorse something is to approve it openly. The structure of the sentence indicates that the part of the sentence after the colon supports or further explains the idea in the first part of the sentence. Although a movie could certainly have an ironic plot (the story could end in an unexpected way, for instance), it does not really make sense to suggest that after knowing what befell the hero, viewers would openly approve of the fate of the villain. Additionally, the viewers endorsement of the villains fate would not be evidence of the movies ironic plot. Choice (E) is incorrect. Spellbinding means holding ones attention as if by a spell. To ignore something is to reject it or refuse to take notice of it. The structure of the sentence indicates that the part of the sentence after the colon supports or further explains the idea in the first part of the sentence. A movie could certainly have a spellbinding plot, holding viewers attention, and it might make sense to suggest that viewers could ignore what happens to the villain once they know what happens to the hero. But this refusal to take notice of the fate of the villain would indicate that the movies plot was not spellbinding: at least part of the plot would not hold viewers attention. 2 Explanation for Correct Answer B. Choice (B) is correct. In this context, to contaminate something is to make it unfit for use by the addition of unwholesome or undesirable elements. To backfire is to have the reverse of the desired or expected effect. The sentence suggests that although the additive put in gasoline is a well-intentioned [fix] to reduce

pollution, the additive is actually having some unintended, undesired effect. If the additive is contaminating the groundwatermaking it unfitits use has backfired; the additive may be successful in reducing one type of pollution (air pollution), but it is increasing another type of pollution (water pollution). Choice (A) is incorrect. To liquefy is to become a liquid. To founder is to give way or collapse. The sentence suggests that although the additive put in gasoline is a well-intentioned [fix] to reduce pollution, the additive is actually having some unintended, undesired effect. A plan to reduce pollution could founder, or collapse, but it does not make sense to say that the additive in the gasoline is liquefying groundwater (water is already a liquid, so the additive could not cause it to become a liquid) or that this is an undesirable result. Choice (C) is incorrect. To purify something is to make it pure or free it from undesirable elements. Something that boomerangs backfires or turns back on its originator. The sentence suggests that although the additive put in gasoline is a well-intentioned [fix] to reduce pollution, the additive is actually having some unintended, undesired effect. Although a plan to reduce pollution could certainly boomerang, or backfire, it is illogical to suggest that making the groundwater pure is an undesired effect. On the contrary, the additives effectsreducing air pollution and purifying waterwould likely be considered desirable. Choice (D) is incorrect. To saturate something is to treat or furnish it with something else to the point where no more can be absorbed or retained. To reciprocate is to give back or make a return for something. The sentence suggests that although the additive put in gasoline is a well-intentioned [fix] to reduce pollution, the additive is actually having some unintended, undesired effect. It is possible that the additive is filling the water with chemicals, so the term saturating might make sense, but the term reciprocate does not logically complete the sentence. It does not make much sense to suggest that a plan can reciprocate, or make a return. Choice (E) is incorrect. To pollute something is to make it physically impure or unclean. To prevail is to be or become effective. The sentence suggests that although the additive put in gasoline is a well-intentioned [fix] to reduce pollution, the additive is actually having some unintended, undesired effect. If the additive is polluting the water it is certainly having an undesired effect, but this pollution does not suggest that the plan to reduce pollution is prevailing, or becoming effective. On the contrary, the measure meant to reduce pollution is now causing pollution. 3 Explanation for Correct Answer A. Choice (A) is correct. The structure of the sentence indicates that the missing term will be similar in meaning to the phrase devoid of any emotion or personal

prejudice. The term dispassionate logically completes the sentence; to be dispassionate is to be unaffected by personal or emotional involvement. Choice (B) is incorrect. The structure of the sentence indicates that the missing term will be similar in meaning to the phrase devoid of any emotion or personal prejudice. The term insubstantial does not have anything to do with being devoid of emotion or prejudice; to be insubstantial is to lack substance or solidity. Choice (C) is incorrect. The structure of the sentence indicates that the missing term will be similar in meaning to the phrase devoid of any emotion or personal prejudice. The term esoteric does not have anything to do with being devoid of emotion or prejudice; to be esoteric is to be understood by only a small, specific group of people. Choice (D) is incorrect. The structure of the sentence indicates that the missing term will be similar in meaning to the phrase devoid of any emotion or personal prejudice. The term capricious does not have anything to do with being devoid of emotion or prejudice; to be capricious is to tend to do things impulsively. Choice (E) is incorrect. The structure of the sentence indicates that the missing term will be similar in meaning to the phrase devoid of any emotion or personal prejudice. The term indignant does not describe someone or something that is devoid of emotion; on the contrary, to be indignant is to be filled with anger because something is unjust, unworthy, or mean. If the biologists description was indignant it would have been charged with emotion, not completely free of it. 4 Explanation for Correct Answer B. Choice (B) is correct. Something that is tenable can be maintained or defended. A misconception is a mistaken idea or notion. The sentence reveals that there are modern Taino descendants in Puerto Rico; therefore, the belief that all Taino people have perished would be a misconception, or a mistaken belief. Additionally, the structure of the sentence suggests that the part of the sentence before the comma describes the belief that all of Puerto Ricos indigenous Taino people perished centuries ago. Because this belief is a misconception, it would not be considered tenable, or able to be defended. Choice (A) is incorrect. Something that is conclusive is decisive or brings an end to a debate. In this context, a reality is a real state of affairs or a real event. The sentence reveals that there are modern Taino descendants in Puerto Rico; therefore, the belief that all Taino people have perished does not reflect the real state of affairs it is not a reality. Additionally, the structure of the sentence suggests that the part of the sentence before the comma describes the belief that all of Puerto Ricos indigenous Taino people perished centuries ago. Although a belief that is found to be untrue would not be considered conclusive, this belief could not also be a reality.

Choice (C) is incorrect. Something that is mythical is fictitious or imaginary. A possibility is something that is possible, or capable of happening or being true. The sentence reveals that although some people think all Taino people have perished, modern Taino descendants have been identified in Puerto Rico. Therefore, the term that fits the second blank should describe a belief that has been found to be untrue; the term possibility does not make sense because it describes something capable of being true. Additionally, the structure of the sentence suggests that the part of the sentence before the comma describes the belief that all of Puerto Ricos indigenous Taino people perished centuries ago. Although one might believe in something that is fictitious, it does not make sense to say that the belief itself was at any time mythical. Choice (D) is incorrect. Something that is erroneous contains or is characterized by error. A delusion is a false belief. The sentence reveals that there are modern Taino descendants in Puerto Rico; therefore, the belief that all Taino people have perished could be described as a delusion, because it is a false belief. But the structure of the sentence suggests that the part of the sentence before the comma describes the belief that all of Puerto Ricos indigenous Taino people perished centuries ago. If the belief has been found to be untrue, it does not make sense to describe it as no longer considered erroneousthis would mean that the belief is now considered correct. Choice (E) is incorrect. Something that is hypothetical involves a hypothesis, or a tentative assumption. A digression is an instance of turning away from a main subject of attention. The sentence reveals that there are modern Taino descendants in Puerto Rico; the belief that all Taino people have perished could have been called hypothetical, because it was an assumption, but it does not make sense to say that the belief appears to be hypothetical now that it has been found to be untrue. Additionally, the structure of the sentence suggests that the part of the sentence before the comma describes the belief that all of Puerto Ricos indigenous Taino people perished centuries ago. It does not make sense to say that the belief was once considered a digression; the sentence does not indicate that any main subject has been turned away from. 5 Explanation for Correct Answer E. Choice (E) is correct. To mollify someone is to soothe his or her temper or disposition. The term Although suggests a contradiction between the first and second parts of the sentence. That is, the mother is easily angered, but the childrens remorse undoes that anger. It makes sense to suggest that the childrens expressions of regret mollified their motherthey soothed her temper. Choice (A) is incorrect. To be substantiated is to be given form or substance. Although the childrens behavior may have substantiated their mothers anger, it does not make sense to describe the mother herself as being substantiatedgiven formby anything the children did or said. Instead, because the term Although

suggests a contradiction between the first and second parts of the sentence, the term that fits the blank should describe someone whose anger has been undone. The term mollified describes such a person. Choice (B) is incorrect. To impugn someone is to attack his or her word as false or lacking integrity. The sentence indicates that the childrens mother is easily angered, but that the childrens expressions of remorse undo that anger. Expressions of remorse are apologies or admissions of guilt, and it does not make sense to suggest that the children impugned their mother with their apologies. It is also illogical to suggest that impugning their mother would undo her anger. Choice (C) is incorrect. To be protected is to be guarded or defended from injury. The term Although suggests a contradiction between the first and second parts of the sentence. That is, the mother is easily angered, but the childrens expressions of remorse undo that angerso the term that fits the blank should describe someone whose anger has been undone. There is no connection between being guarded from injury and having ones anger undone, so the term protected does not logically complete the sentence. Rather, the term mollified describes someone whose anger has been soothed. Choice (D) is incorrect. To be united is to be made one or combined. The term Although suggests a contradiction between the first and second parts of the sentence. That is, the mother is easily angered, but the childrens expressions of remorse undo that angerso the term that fits the blank should describe someone whose anger has been undone. Although it would make sense to say that undoing the mothers anger could unite all of the family members, it does not make sense to say that the mother alone could be unitedshe is a single entity. 6 Explanation for Correct Answer A. Choice (A) is correct. Flotsam is floating debris, especially debris that is miscellaneous or unimportant. The term other indicates that the term that completes the sentence describes dead satellites, jettisoned rockets, and drifting paint flecks that are orbiting Earth. These miscellaneous items are all debris, so it makes sense to label them flotsam. Choice (B) is incorrect. Reconnaissance is a survey to gain information. The term other indicates that the term that completes the sentence describes dead satellites, jettisoned rockets, and drifting paint flecks that are orbiting Earth. It does not make sense to suggest that any one of these items is a survey, so the term reconnaissance does not logically complete the sentence. Rather, the term flotsam describes such debris. Choice (C) is incorrect. Decimation is the process of drastically reducing something in number, especially through harm. The term other indicates that the term that completes the sentence describes dead satellites, jettisoned rockets, and

drifting paint flecks that are orbiting Earth. Although these items could be the result of the drastic reduction and harming of something, it does not make sense to describe the debris itself as decimation. Rather, such debris is best described as flotsam. Choice (D) is incorrect. Raiment is clothing. The term other indicates that the term that completes the sentence describes dead satellites, jettisoned rockets, and drifting paint flecks that are orbiting Earth. These items are not articles of clothing, so the term raiment does not logically complete the sentence. Rather, the term flotsam accurately describes the debris mentioned in the sentence. Choice (E) is incorrect. Sustenance is a means of nourishment or support. The term other indicates that the term that completes the sentence describes dead satellites, jettisoned rockets, and drifting paint flecks that are orbiting Earth. The sentence does not indicate what such debris might be supporting, and it does not make sense to describe the debris as nourishing, so the term sustenance does not logically complete the sentence. Rather, the term flotsam accurately describes the debris mentioned in the sentence. 7 Explanation for Correct Answer A. Choice (A) is correct. In this context, disposition means mood or tendency to act a certain way. Cantankerous means difficult to deal with. The sentence states that although a persons body may change with age (physiological changes), there is something about the person that does not change. The colon indicates that the second half of the sentence further explains the idea presented in the first half by providing an example: a persons irascibility, or tendency to be easily angered, would not change. Therefore, it makes sense to describe an individuals disposition as what stays the same. Further, the use of the term still in the second half of the sentence suggests that the term that fits the second blank is a synonym of the term irascible. Because an irascible person has a hot temper, he or she likely is cantankerous, or difficult to deal with. Choice (B) is incorrect. Anatomy is ones physical makeup, or the composition of ones body parts. Churlish means vulgar or surly. The sentence states that although a persons body changes with age (physiological changes), there is something about the person that does not changesomething that does not have to do with the body. A persons anatomy is his or her body, so the term anatomy does not make sense in the first blank. Additionally, it makes sense to suggest that a thirty-year-old who is irascible, or easily angered, might be surly, but the colon indicates that the second half of the sentence further explains the idea expressed in the first half. The second half of the sentence refers to a persons behavior irascible and churlishrather than his or her physical makeup. Choice (C) is incorrect. An outlook is a perspective or point of view. Benevolent means having goodwill and being disposed to doing good. The

sentence states that although a persons body changes with age (physiological changes), there is something about the person that does not change. The colon indicates that the second half of the sentence further explains the idea presented in the first half by providing an example: a persons irascibility, or tendency to be easily angered, would not change. If what stays the same is a persons irascibility, then it could make sense to describe a persons outlook, or perspective, as staying the same. But the term benevolent does not logically complete the sentence; the use of the term still in the second half of the sentence suggests that the term that fits the second blank is a synonym of the term irascible. An irascible person could also be benevolent, but being hot tempered is not directly related to having goodwill, so the term benevolent is not a synonym of irascible. Choice (D) is incorrect. Personality is ones behavioral and emotional characteristics. Laconic means using few words or being concise to the point of seeming rude. The sentence states that although a persons body changes with age (physiological changes), there is something about the person that does not change. The colon indicates that the second half of the sentence further explains the idea presented in the first half by providing an example: a persons irascibility, or tendency to be easily angered, would not change. If what stays the same is a persons irascibility, it could make sense to describe a persons personality, or emotional qualities, as staying the same. But the term laconic does not logically complete the sentence; the use of the term still in the second half of the sentence suggests that the term that fits the second blank is a synonym of the term irascible. A person who does not speak much could also be irascible, but being hot tempered is not directly related to using few words, so the term laconic is not a synonym of irascible. Choice (E) is incorrect. Stature is ones status gained by growth or achievement. Robust means having strength or vigor. The sentence states that although a persons body changes with age (physiological changes), there is something about the person that does not change. The colon indicates that the second half of the sentence further explains the idea presented in the first half by providing an example: a persons irascibility would not change. Being irascible, or easily angered, is not directly related to having a certain status, so the idea that irascibility does not change is not an example of a persons unchanging stature. Additionally, the use of the term still in the second half of the sentence suggests that the term that fits the second blank is a synonym of the term irascible. A person who has strength could also be irascible, but being hot tempered is not directly related to being strong, so the term robust is not a synonym of irascible. 8 Explanation for Correct Answer A. Choice (A) is correct. The sentence indicates that the electorate was unpredictable and had constantly shifting moods. Because the term mercurial describes

something that is characterized by quick and often erratic mood changes, it makes sense to describe the electorate as mercurial. Choice (B) is incorrect. In this context, corrosive means bitingly sarcastic. The sentence indicates that the electorate was described in a certain way as a result of being unpredictable and having constantly shifting moods. Therefore, the term that fits the blank must describe a group that is unpredictable and experiences mood changes. It is possible that the electorate could be sarcastic when in a certain mood, but the term corrosive does not necessarily describe a group with unpredictable mood changes. Choice (C) is incorrect. Disingenuous means falsely appearing to be frank or candid. The sentence indicates that the electorate was described in a certain way as a result of being unpredictable and having constantly shifting moods. Therefore, the term that fits the blank must describe a group that is unpredictable and experiences mood changes. It is possible that an unpredictable group might be disingenuous at times, but the term disingenuous does not necessarily describe a group with unpredictable mood changes. Choice (D) is incorrect. Implacable means incapable of being appeased or mollified. The sentence indicates that the electorate was described in a certain way as a result of being unpredictable and having constantly shifting moods. Therefore, the term that fits the blank must describe a group that is unpredictable and experiences mood changes. The term implacable is not directly connected to unpredictable mood changes. Indeed, a group that could not be mollified might not have constantly shifting moods, but rather one mood (such as anger), and the groups inability to be mollified might be predictable. Choice (E) is incorrect. Phlegmatic means having a slow and stolid, or unemotional, temperament. The sentence indicates that the electorate was described in a certain way as a result of being unpredictable and having constantly shifting moods. Therefore, the term that fits the blank must describe a group that is unpredictable and experiences mood changes. The term phlegmatic is not directly connected to unpredictable mood changes. If anything, such mood changes suggest that the electorate is emotional, not unemotional. 9 Explanation for Correct Answer D. Choice (D) is correct. When considering the issue of overpopulation, the author of Passage 1 discusses the worlds population relative to the land available. The author of Passage 2, however, believes that the key issue in judging overpopulation is not how many people can fit into any given space but whether Earth can supply the populations long-term requirement for food, water, and other resources. He or she would likely criticize the author of Passage 1 for focusing on the wrong factor in considering the issue of overpopulation.

Choice (A) is incorrect. The author of Passage 1 considers the worlds population relative to the land available and presents relevant data to support his or her claims. The author of Passage 2 concentrates not on the number of people per square mile, but whether Earth can supply the populations long-term requirement for food, water, and other resources. The author of Passage 2 says that the idea presented in Passage 1 "is . . . wrong," so he or she might consider it a flawed conclusion; but the author of Passage 2 does not indicate that those who consider the worlds population relative to the land available are using incorrect data. Choice (B) is incorrect. The author of Passage 1 considers the worlds population relative the land available and makes the claim that the world is actually underpopulated. In other words, the world does not have an overpopulation problem. The author of Passage 1 is not acknowledging overpopulation as a global problem at all; therefore, it does not make sense to suggest that the author of Passage 2 would criticize him or her for severely overstating this problem. Choice (C) is incorrect. The author of Passage 1 considers the worlds population relative to the land available and presents relevant data to support his or her claim that the world is actually underpopulated. Passage 1 does not include any recommendations for a course of action that might have damaging effects. In fact, the author does not even acknowledge overpopulation as a problem, so it is unlikely that he or she would offer any possible solutions or courses of action. Choice (E) is incorrect. The author of Passage 1 considers the worlds population relative to the land available and presents relevant data to support his or her claim that the world is actually underpopulated. All of the data provided is based on the work of a noted economist. There is no discussion of any personal issues or prejudices in relation to this scientific inquiry. 10 Explanation for Correct Answer C. Choice (C) is correct. The tone of the first sentence of Passage 2 is best characterized as emphaticuttered with or marked by emphasis. The author of Passage 2 uses this sentence to emphasize, or stress the importance of, the fact that a certain idea is wrong; he or she states forcefully that this idea is as widespread as it is wrong." This is an emphatic opening sentence. Choice (A) is incorrect. The author of Passage 2 uses the first sentence to emphasize, or stress the importance of, the fact that one approach to the issue of overpopulationfocusing on the number of people per square mileis wrong. By immediately pointing out the error in this approach, the author is strongly insisting that his or her way of thinking about overpopulation is the correct way. Therefore, it is not logical to describe the tone of this sentence as wistful, or sad or melancholy.

Choice (B) is incorrect. The author of Passage 2 uses the first sentence to emphasize, or stress the importance of, the fact that one approach to the issue of overpopulationfocusing on the number of people per square mileis wrong. By immediately pointing out the error in this approach, the author is strongly insisting that his or her way of thinking about overpopulation is the correct way. To be dismayed is to lose courage or resolution. The first sentence does not suggest that the author is dismayed; on the contrary, the sentence indicates that he or she is confident in his or her views regarding overpopulation. Choice (D) is incorrect. The author of Passage 2 uses the first sentence to emphasize, or stress the importance of, the fact that one approach to the issue of overpopulationfocusing on the number of people per square mileis wrong. By immediately pointing out the error in this approach, the author is strongly insisting that his or her way of thinking about overpopulation is the correct way. In this context, to be ambivalent is to be uncertain as to which approach to follow. The first sentence does not suggest that the author is ambivalent; on the contrary, the sentence indicates that he or she is certain that his or her approach is accurate. Choice (E) is incorrect. The author of Passage 2 uses the first sentence to emphasize, or stress the importance of, the fact that one approach to the issue of overpopulationfocusing on the number of people per square mileis wrong. By immediately pointing out the error in this approach, the author is strongly insisting that his or her way of thinking about overpopulation is the correct way. To be apologetic is to regretfully acknowledge fault or failure. The first sentence does not suggest that the author is apologetic; on the contrary, the sentence indicates that he or she believes that the fault or failure lies with those who focus on the number of people per square mile. 11 Explanation for Correct Answer C. Choice (C) is correct. The author of Passage 1 claims that the world is filled with empty places and mentions vast expanses of unoccupied land. According to the author of Passage 2, most land that people think of as empty either grows the food essential to our well-being or supplies us with raw materials. These places may not be occupied by people, but they are filled with life-sustaining resources; they are not truly empty. The author of Passage 2 uses quotation marks around the word empty to express his or her disagreement with the type of characterization found in Passage 1. Choice (A) is incorrect. The author of Passage 1 claims that the world is filled with empty placesvast expanses of unoccupied landand says that when we look at the worlds population relative to the land available, we find out just how underpopulated the world is. The author of Passage 2 would likely say the author Passage 1 is understating or discounting the problem of overpopulation, not overstating it. Therefore, it does not make sense to say that the author of Passage 2

uses quotation marks around the word empty to criticize Passage 1 for overstating the nature of a problem. Choice (B) is incorrect. There is no discussion in either passage of a change in population patterns; the passages present contrasting views regarding the issue of overpopulation. The author of Passage 1 claims that the world is filled with empty places and mentions vast expanses of unoccupied land. According to the author of Passage 2, the land that people think of as empty is not truly empty; most of this land either grows the food essential to our well-being or supplies us with raw materials. The author of Passage 2 uses quotation marks around the word empty to express his or her disagreement with the type of characterization found in Passage 1. Choice (D) is incorrect. The author of Passage 1 claims that the world is filled with empty placesvast expanses of unoccupied landand says that when we look at the worlds population relative to the land available, we find out just how underpopulated the world is. The author is not advancing a problem; rather, he or she is essentially refusing to acknowledge that overpopulation is a problem. Further, the author of Passage 2 offers no solution to the problem of overpopulation; he or she merely presents a different way of considering the issue. Choice (E) is incorrect. The author of Passage 1 claims that the world is filled with empty places and mentions vast expanses of unoccupied land. According to the author of Passage 2, the land that people think of as empty is not truly empty; most of this land either grows the food essential to our well-being or supplies us with raw materials. The author of Passage 2 uses quotation marks around the word empty to express his or her disagreement with the type of characterization found in Passage 1. The author of Passage 2 does not concur, or agree, with any theories mentioned in Passage 1. 12 Explanation for Correct Answer A. Choice (A) is correct. The author of Passage 1 states that the world is filled with empty places and mentions vast expanses of unoccupied land. The author of Passage 2 also acknowledges that there is much empty land, though he or she maintains this land is not truly empty (it either grows the food essential to our well-being or supplies us with raw materials), and refers to unoccupied areas when he or she states that densely populated countries and cities can be crowded only because the rest of the world is not. Choice (B) is incorrect. The author of Passage 1 refers to the worlds six billion people but does not comment on the reliability of this estimate. Further, the author of Passage 2 does not mention anything about estimates of Earths population. Choice (C) is incorrect. The author of Passage 1 does not discuss the worlds empty spaces in terms of their productivity. He or she focuses on the worlds

population relative to the land available and concludes that the world is underpopulated. Although the author of Passage 2 does discuss the empty spaces as productive land (Most of the empty land in the United States . . . either grows the food essential to our well-being or supplies us with raw materials), he or she does not mention the role of technology in the lands productivity. Choice (D) is incorrect. The authors clearly differ in their views regarding the dangers of overpopulation. The author of Passage 1 does not acknowledge overpopulation as a problemhe or she even claims that the world is underpopulatedwhile the author of Passage 2 seems to appreciate the dangers of overpopulation. Further, neither author specifically mentions scientists or nonscientists (the author of Passage 1 only mentions a noted economist), and neither makes any judgments regarding nonscientists views of the issue of overpopulation. Choice (E) is incorrect. The author of Passage 2 does discuss the idea that Earths population could outstrip, or exceed, available resources: he or she states that the key issue in judging overpopulation is . . . whether Earth can supply the populations long-term requirement for food, water, and other resources. The author of Passage 1, however, does not focus on the Earths population relative to available resources. Instead, he or she considers the worlds population relative to the land available, or how many people can fit into any given space. 13 Explanation for Correct Answer B. Choice (B) is correct. In lines 1-5, the narrator uses language that describes actions and conditions normally associated with animate, or living, beings. A map is certainly not an animate object, but it is described as if it is a living thing: the map is forced to show its colors and is described as curling and snapping back. The narrator also states that the whole land was very tense until we put our four steins on its corners and laid the river out. Choice (A) is incorrect. Invaluable means priceless, or valuable beyond estimation. A map is certainly a valuable thing to have when one is embarking on a canoe trip, as the four men are preparing to do as this passage begins. But the narrators descriptions in lines 1-5 do not focus on the importance or value of the map. The narrator is simply describing the mapthe colors, the land, the river, the mountainsas the four men unroll it and begin to examine it in anticipation of their trip. Choice (C) is incorrect. Cryptic means mysterious, or having a hidden or ambiguous meaning. In lines 1-5, the narrator is simply describing the mapthe colors, the land, the river, the mountainsas the four men unroll it and begin to examine it in preparation for their canoe trip. The men do not seem confused by the map, as one might expect if the map had some hidden or ambiguous meaning.

Therefore, it does not make sense to suggest that the map is being described as cryptic, or mysterious. Choice (D) is incorrect. Antiquated means obsolete, or no longer useful. In lines 1-5, the narrator is simply describing the mapthe colors, the land, the river, the mountainsas the four men unroll it and begin to examine it in preparation for their canoe trip. There is no indication that the map is antiquated in any way. If the map were antiquated, or no longer useful, then it is unlikely that the men would be examining it and planning to use it for their trip. Choice (E) is incorrect. Erroneous means containing or characterized by error. In lines 15, the narrator is simply describing the mapthe colors, the land, the river, the mountains as the four men unroll it and begin to examine it in preparation for their canoe trip. There is nothing to suggest that the map is erroneous. If the map were erroneous, or contained errors or mistakes, then it is unlikely that the men would be examining it and planning to use it for their trip. 14 Explanation for Correct Answer C. Choice (C) is correct. An impression is an effect, feeling, or belief retained as a consequence of an experience. In lines 9-14, the narrator describes observing Lewis hand as it moves across the map and listening as Lewis speaks about the map. From this experience, the narrator gains a strong impression of Lewis' power and leadership qualities. The narrator notes that Lewis hand seemed to have power over the terrain, and that, when Lewis explained something, it was as though all streams everywhere quit running, hanging silently where they were to let the point be made. These lines primarily serve to note the narrators impression of Lewis, the feelings and beliefs about Lewis that the narrator developed as a result of this experience.. Choice (A) is incorrect. An anecdote is a short narrative or description of events. Although lines 9-14 refer to an interesting experience from the narrators life, they are not best described as recounting an anecdote. In these lines, the narrator focuses on an impression rather than on a series of events: the fact that Lewis hand seemed to have power over the terrain, the effect that Lewis seemed to have on the landscape when he stopped to explain something (it was as though all streams everywhere quit running, hanging silently where they were to let the point be made). These lines are primarily intended to convey the narrators impression of Lewis, not to tell a story.. Choice (B) is incorrect. In lines 9-14, the narrator describes observing Lewis hand as it moves across the map and listening as Lewis speaks about the map. From this experience, the narrator gains a strong impression of Lewis' power and leadership qualities. The narrator notes that Lewis hand seemed to have power over the terrain, and that, when Lewis explained something, it was as though all streams everywhere quit running, hanging silently where they were to let the point be made. These lines are intended to

convey the narrators impression of Lewis as a result of this experience; the narrator is not offering an example of anything.. Choice (D) is incorrect. In lines 9-14, the narrator describes observing Lewis hand as it moves across the map and listening as Lewis speaks about the map. From this experience, the narrator gains a strong impression of Lewis' power and leadership qualities. The narrator notes that Lewis hand seemed to have power over the terrain, and that, when Lewis explained something, it was as though all streams everywhere quit running, hanging silently where they were to let the point be made. These lines are intended to convey the narrators impression of Lewis as a result of this experience. The narrator does not make a prediction, or declare in advance that something specific will happen in the future.. Choice (E) is incorrect. In lines 9-14, the narrator describes observing Lewis hand as it moves across the map and listening as Lewis speaks about the map. From this experience, the narrator gains a strong impression of Lewis' power and leadership qualities. The narrator notes that Lewis hand seemed to have power over the terrain, and that, when Lewis explained something, it was as though all streams everywhere quit running, hanging silently where they were to let the point be made. The narrator does not propose a theory, or an unproven assumption, of any kind; rather, he conveys his impression of Lewis as a result of an experience. 15 Explanation for Correct Answer D. Choice (D) is correct. Omnipotent means all-powerful, or having virtually unlimited authority or influence. The narrator states that Lewis hand seemed to have power over the terrain. Lewis hand is so powerful that it seems to control the landscape represented by the map: when Lewis' hand stopped for Lewis voice to explain something, it was as though all streams everywhere quit running, hanging silently where they were to let the point made.. Choice (A) is incorrect. Deft means characterized by facility and skill. The narrator is not necessarily suggesting that Lewis hand is skillful. While Lewis hand may be skillful as it took a pencil and . . . began to work downstream, the narrator is concentrating on Lewis hand because it seemed to have power over the terrain. The narrator suggests that the power in Lewis hand has a dramatic effect on the landscape: when Lewis hand stopped for Lewis voice to explain something, it was as though all streams everywhere quit running.. Choice (B) is incorrect. Languid means weak or sluggish. The narrator states that Lewis hand seemed to have power over the terrain. He goes on to suggest that the power in Lewis hand has a dramatic effect on the landscape: when Lewis hand stopped for Lewis voice to explain something, it was as though all streams everywhere quit running. It does not make sense to suggest that a languid, or weak, hand would have such a powerful effect..

Choice (C) is incorrect. Resilient means tending to recover from or adjust easily to misfortune or change. The narrator states that Lewis hand seemed to have power over the terrain. He goes on to suggest that the power in Lewis hand has a dramatic effect on the landscape: when Lewis hand stopped for Lewis voice to explain something, it was as though all streams everywhere quit running. The only change that seems to take place the flow of the streams stoppingis caused by the apparent power of Lewis hand, so there is no need for the hand to recover from or adjust to it.. Choice (E) is incorrect. Expressive means effectively conveying meaning or feeling. The narrator states that Lewis hand seemed to have power over the terrain. Lewis hand is so powerful that it seems to have an effect on the landscape: when Lewis hand stopped for Lewis voice to explain something, it was as though all streams everywhere quit running. Lewis hand may have some expressive qualities, but the focus of these lines is on the hand's apparent power to affect the landscape. 16 Explanation for Correct Answer E. Choice (E) is correct. In line 13, the word hanging is used to describe streams that have stopped flowing. According to the narrator, Lewis hand is so powerful that when it stops to allow Lewis to explain something, it was as though all streams everywhere quit running, hanging silently where they were to let the point be made. Something that is suspended is stopped temporarily; when Lewis hand stops, the streams temporarily stop running.. Choice (A) is incorrect. The word flowing means moving in a stream. In line 13, the word hanging is used to describe streams that have stopped running, or moving. According to the narrator, Lewis hand is so powerful that when it stops to allow Lewis to explain something, it was as though all streams everywhere quit running, hanging silently where they were to let the point be made. Streams that have quit moving cannot be described as flowing, or moving; in this context, the words hanging and flowing are opposite in meaning.. Choice (B) is incorrect. Drooping means sinking gradually or inclining downward. In line 13, the word hanging is used to describe streams that have stopped flowing. According to the narrator, Lewis hand is so powerful that when it stops to allow Lewis to explain something, it was as though all streams everywhere quit running, hanging silently where they were to let the point be made. The narrator states that the streams have quit running, so it makes more sense to say that the streams are "suspended" than that the streams are drooping. The word drooping suggests that there is a sinking movement taking place, whereas the passage indicates that movement in the stream has come to a halt.. Choice (C) is incorrect. Inclining means leaning, slanting, or sloping. In line 13, the word hanging is used to describe streams that have stopped running flowing. According to the

narrator, Lewis hand is so powerful that when it stops to allow Lewis to explain something, it was as though all streams everywhere quit running, hanging silently where they were to let the point be made. The streams have quit running, so it does not make sense to say that the streams are inclining. The word inclining indicates that there is a leaning or slanting movement taking place, but the passage indicates that movement in the stream has come to a halt.. Choice (D) is incorrect. Unfinished means not finished or incomplete. In line 13, the word hanging is used to describe streams that have stopped flowing. According to the narrator, Lewis hand is so powerful that when it stops to allow Lewis to explain something, it was as though all streams everywhere quit running, hanging silently where they were to let the point be made. In this context, the word hanging is used to describe the streams lack of movement. There is no connection between the movement of a stream and its incompleteness; further, it does not make sense to describe a stream as unfinished. 17 Explanation for Correct Answer E. Choice (E) is correct. As Lewis describes a valley that will eventually be under water, he states, But right now its wild. . . . it looks like something up in Alaska. We really ought to go up there before the real estate people get hold of it and make it over into one of their heavens. Lewis implies that, in Alaska, the real estate people have not yet built homes or businesses on the land. Lewis mentions Alaska as an example of an undeveloped place, a wild place where the land has not been made suitable for working or living.. Choice (A) is incorrect. As Lewis describes a valley that will eventually be under water, he states, But right now its wild. . . . it looks like something up in Alaska. We really ought to go up there before the real estate people get hold of it and make it over into one of their heavens. Lewis implies that, in Alaska, the real estate people have not yet built homes or businesses on the land: it is wild and undeveloped. There is no mention of how distant, or far away, the valley is from their current location.. Choice (B) is incorrect. As Lewis describes a valley that will eventually be under water, he states, But right now its wild. . . . it looks like something up in Alaska. We really ought to go up there before the real estate people get hold of it and make it over into one of their heavens. The valley, which is around fifty miles long, might be considered immense, or extremely large, and Alaska is a large state. But in these lines Lewis simply implies that, in Alaska, the real estate people have not yet built homes or businesses on the land: it is wild and undeveloped. He is eager to visit the valley because it is wild, not because it is large.. Choice (C) is incorrect. As Lewis describes a valley that will eventually be under water, he states, But right now its wild. . . . it looks like something up in Alaska. We really ought to go up there before the real estate people get hold of it and make it over into one of their heavens. Lewis implies that, in Alaska, the real estate people have not yet built homes

or businesses on the land: it is wild and undeveloped. There is no mention of anything scenic, or relating to natural scenery, about the valley.. Choice (D) is incorrect. As Lewis describes a valley that will eventually be under water, he states, But right now its wild. . . . it looks like something up in Alaska. We really ought to go up there before the real estate people get hold of it and make it over into one of their heavens. Lewis implies that, in Alaska, the real estate people have not yet built homes or businesses on the land: it is wild and undeveloped. Lewis does not mention the weather in either Alaska or the valley, so it does not make sense to suggest that Alaska is used as an example of a cold place. 18 Explanation for Correct Answer A. Choice (A) is correct. A person who is contemptuous has a lack of respect or reverence for someone or something. Lewis commentWe really ought to get up there before the real estate people get hold of it and make it over into one of their heavensimplies that the real estate people do not necessarily make a place better by building homes, businesses, and other pieces of property on it. Because Lewis wants to visit the land before the real estate people get hold of it, it is logical to conclude that Lewis does not have any respect for them or what they do.. Choice (B) is incorrect. Lewis commentWe really ought to get up there before the real estate people get hold of it and make it over into one of their heavensimplies that the real estate people do not necessarily make a place better by building homes, businesses, and other pieces of property on it. Because Lewis wants to visit the land before the real estate people get hold of it, it is logical to conclude that Lewis does not have any respect for them or what they do. To be envious means to be jealous or resentful of anothers advantages. Because Lewis has no respect for the real estate people, it is unlikely that he would be envious of them.. Choice (C) is incorrect. Lewis commentWe really ought to get up there before the real estate people get hold of it and make it over into one of their heavensimplies that the real estate people do not necessarily make a place better by building homes, businesses, and other pieces of property on it. Because Lewis wants to visit the land before the real estate people get hold of it, it is logical to conclude that Lewis does not have any respect for them or what they do. Although he may not respect them, Lewis is not conveying a furious, or extremely angry, attitude.. Choice (D) is incorrect. Lewis commentWe really ought to get up there before the real estate people get hold of it and make it over into one of their heavensimplies that the real estate people do not necessarily make a place better by building homes, businesses, and other pieces of property on it. Because Lewis wants to visit the land before the real estate people get hold of it, it is logical to conclude that Lewis does not have any respect for them or what they do. Although he may not respect them, there is no indication that Lewis is puzzled, or perplexed and confused, by the real estate people..

Choice (E) is incorrect. Lewis commentWe really ought to get up there before the real estate people get hold of it and make it over into one of their heavensimplies that the real estate people do not necessarily make a place better by building homes, businesses, and other pieces of property on it. Because Lewis wants to visit the land before the real estate people get hold of it, it is logical to conclude that Lewis does not have any respect for them or what they do. Lewis is certainly not intrigued, or curious and interested, by the real estate people. On the contrary, he does not want anything to do with them or their efforts to develop the land. 19 Explanation for Correct Answer E. Choice (E) is correct. The word heaven refers to a place or condition of utmost happiness. Lewis suggests that a place that is fully developed with homes, businesses, and other pieces of property that can be bought and sold is heaven for the real estate people. His commentWe should go up there before the real estate people get hold of it and make it over into one of their heavensimplies that he does not agree with the real estate peoples concept of heaven. If he did, then he would wait until after the real estate people changed the land to visit. Therefore, by using the word heavens in this context, Lewis is being ironic; he is using the word to express something that is the opposite of the literal meaning.. Choice (A) is incorrect. The word heaven refers to a place or condition of utmost happiness. Lewis suggests that a place that is fully developed with homes, businesses, and other pieces of property that can be bought and sold is heaven for the real estate people. His commentWe should go up there before the real estate people get hold of it and make it over into one of their heavensimplies that he does not agree with the real estate peoples concept of heaven. If he did, then he would wait until after the real estate people changed the land to visit. Lewis is using the word heavens ironically, to show his disagreement with the views of the real estate people, not to show appreciation for them.. Choice (B) is incorrect. The word heaven refers to a place or condition of utmost happiness. Lewis suggests that a place that is fully developed with homes, businesses, and other pieces of property that can be bought and sold is heaven for the real estate people. His commentWe should go up there before the real estate people get hold of it and make it over into one of their heavensimplies that he does not agree with the real estate peoples concept of heaven. If he did, then he would wait until after the real estate people changed the land to visit. Lewis is using the word heavens ironically, to show his disagreement with the views of the real estate people. His meaning is clear; he is not intending to deceive or mislead anyone.. Choice (C) is incorrect. The word heaven refers to a place or condition of utmost happiness. Lewis suggests that a place that is fully developed with homes, businesses, and other pieces of property that can be bought and sold is heaven for the real estate people. His commentWe should go up there before the real estate people get hold of it

and make it over into one of their heavensimplies that he does not agree with the real estate peoples concept of heaven. If he did, then he would wait until after the real estate people changed the land to visit. Therefore, Lewis is using the word heavens ironically, to show his disagreement with the views of the real estate people. He is certainly not tentativehesitant or uncertainabout his viewpoint.. Choice (D) is incorrect. The word heaven refers to a place or condition of utmost happiness. Lewis suggests that a place that is fully developed with homes, businesses, and other pieces of property that can be bought and sold is heaven for the real estate people. His commentWe should go up there before the real estate people get hold of it and make it over into one of their heavensimplies that he does not agree with the real estate peoples concept of heaven. Lewis is using the word heavens ironically, to show his disagreement with the views of the real estate people. In this context, defensive means protecting oneself from criticism. Lewis believes his viewpoint is correct; he is not trying to protect himself from others criticism by using the word "heavens." 20 Explanation for Correct Answer B. Choice (B) is correct. In the second paragraph, Lewis mentions that a dam is being built and then describes how the land will change when the dam is completed. His description causes the narrator to try to see the changes that would come, to think about specific ways the dam will create a drastic change in the land as Lewis said it was at this moment. In lines 25-30, the narrator is imagining two radically different states: the land as it is now and the land after the dam is finished.. Choice (A) is incorrect. In the second paragraph, Lewis mentions that a dam is being built and then describes how the land will change when the dam is completed. His description causes the narrator to try to see the changes that would come, to think about specific ways the dam will create a drastic change in the land as Lewis said it was at this moment. In lines 25-30, the narrator is visualizing a series of events, but it is not an unlikely series of events. The passage depicts a clear cause-and-effect relationship; it is very likely that the building of a dam will change the landscape as the narrator visualizes.. Choice (C) is incorrect. In the second paragraph, Lewis mentions that a dam is being built and then describes how the land will change when the dam is completed. His description causes the narrator to try to see the changes that would come, to think about specific ways the dam will create a drastic change in the land as Lewis said it was at this moment. It is possible that the building of the dam is a solution to some problem, but no specific problems or solutions are mentioned anywhere in the passage. Rather, in lines 25-30, the narrator is imagining two radically different states: the land as it is now and the land after the dam is finished.. Choice (D) is incorrect. In the second paragraph, Lewis mentions that a dam is being built and then describes how the land will change when the dam is completed. His description causes the narrator to try to see the changes that would come, to think about specific

ways the dam will create a drastic change in the land as Lewis said it was at this moment. The building of the dam might be considered a course of action by some, but the narrator does not refer to it as such and does not consider the pros and cons of the action. Rather, in lines 25-30, the narrator is simply imagining both the land as it is now and the land after the dam is finished.. Choice (E) is incorrect. In the second paragraph, Lewis mentions that a dam is being built and then describes how the land will change when the dam is completed. His description causes the narrator to try to see the changes that would come, to think about specific ways the dam will create a drastic change in the land as Lewis said it was at this moment. The narrators visualization in lines 25-30 is related to the way the future will be different from the present: when the dam is built the land that is now unvisited and free will become a new lake. But the narrator does not consider or comment on events in the past and how they shape the future. 21 Explanation for Correct Answer A. Choice (A) is correct. In the first half of the passage, Lewis is describing an area of wilderness that will soon be covered in water because a dam is being built. After his description, he suggests that the group take a trip to this area: We really ought to go up there before the real estate people get hold of it . . . . In response to Lewis statement, the narrator realizes that his body, particularly the back and arms, felt ready for something like this. In other words, the narrator is able to imagine the trip through his bodyhe is anticipating the trip not just mentally, but also physically.. Choice (B) is incorrect. In the first half of the passage, Lewis is describing an area of wilderness that will soon be covered in water because a dam is being built. After his description, he suggests that the group take a trip to this area: We really ought to go up there before the real estate people get hold of it . . . . In response to Lewis statement, the narrator realizes that his body, particularly the back and arms, felt ready for something like this. The narrator understands that the trip may be mentally and physically challenging because Lewis liked extremely specialized and difficult form[s] of sport, but he does not express any insecurities about his athletic abilities. Rather, the feeling of readiness implies that the narrator is confident in his abilities.. Choice (C) is incorrect. In the first half of the passage, Lewis is describing an area of wilderness that will soon be covered in water because a dam is being built. After his description, he suggests that the group take a trip to this area: We really ought to go up there before the real estate people get hold of it . . . . In response to Lewis statement, the narrator realizes that his body, particularly the back and arms, felt ready for something like this. Rather than indicating that he is experiencing discomfort, the narrators thought indicates that he is ready for the trip; he is anticipating the trip not just mentally, but also physically..

Choice (D) is incorrect. In the first half of the passage, Lewis is describing an area of wilderness that will soon be covered in water because a dam is being built. After his description, he suggests that the group take a trip to this area: We really ought to go up there before the real estate people get hold of it . . . . In response to Lewis statement, the narrator realizes that his body, particularly the back and arms, felt ready for something like this. Nothing in the third paragraph suggests that the narrator is thinking about his personal appearance, or the way his back and arms look. Rather, the narrators thought indicates that he is ready for the trip; he is anticipating the trip not just mentally, but also physically.. Choice (E) is incorrect. In the first half of the passage Lewis is describing an area of wilderness that will soon be covered in water because a dam is being built. After his description, he suggests that the group take a trip to this area: We really ought to go up there before the real estate people get hold of it . . . . In response to Lewis statement, the narrator realizes that his body, particularly the back and arms, felt ready for something like this. Nothing in the third paragraph suggests that the narrator is thinking about contests of strength or his own success. Rather, the narrators thought indicates that he is ready for the trip; he is anticipating the trip not just mentally, but also physically. 22 Explanation for Correct Answer A. Choice (A) is correct. In the first half of the passage, Lewis describes the land represented on a map and traces the path of a river on that map: he marked out a small strong X . . . and began to work downstream . . . through the printed woods. In line 34, the phrase picking up means finding or locating. When the narrator looks at the map, picking up the river where [they] would enter it, he is locating on the map where the group will enter the river when they take their trip.. Choice (B) is incorrect. In the first half of the passage, Lewis describes the land represented on a map and traces the path of a river on that map: he marked out a small strong X . . . and began to work downstream . . . through the printed woods. To acquire something is to get or possess it. When the narrator looks at the map, picking up the river where [they] would enter it, he does not actually come to possess or own the river. Rather, he locates on the map where the group will enter the river when they take their trip.. Choice (C) is incorrect. In the first half of the passage, Lewis describes the land represented on a map and traces the path of a river on that map: he marked out a small strong X . . . and began to work downstream . . . through the printed woods. The phrase picking up can refer to learning, but in line 34, it is not used to suggest that the narrator is learning or acquiring new information about the river. Rather, the narrator is viewing and locating on the map where the group will enter the river when they take their trip.. Choice (D) is incorrect. In the first half of the passage, Lewis describes the land represented in a map and traces the path of a river on that map: he marked out a small strong X . . . and began to work downstream . . . through the printed woods. To claim

something is to assert ones ownership of it. When the narrator looks at the map, picking up the river where [they] would enter it, he does not assert ownership of the river. Rather, he looks at the map to locate where the group will enter the river when they take their trip.. Choice (E) is incorrect. In the first half of the passage, Lewis describes the land represented on a map and traces the path of a river on that map: he marked out a small strong X . . . and began to work downstream . . . through the printed woods. To gather is to bring together or collect. When the narrator looks at the map, picking up the river where [they] would enter it, he does not bring anything together or collect anything. Rather, he looks at the map to locate where the group will enter the river when they take their trip. 23 Explanation for Correct Answer D. Choice (D) is correct. Someone who is didactic makes moral observations and teaches others. In lines 39-40, the narrator recognizes that Lewis is likely going to turn the trip into A lesson. A moral. A life principle. A Way. The fact that the narrator thinks Lewis will turn the trip into a lesson and a moral suggests that Lewis has previously turned other things into lessons and morals. Indeed, the narrator goes on to say that Lewis liked . . . to take some extremely specialized and difficult form of sport . . . and evolve a personal approach to it which he could then expound. These statements indicate that Lewis is sometimes didactic.. Choice (A) is incorrect. Someone who is whimsical is subject to erratic behavior or unpredictable change. In lines 39-40, the narrator recognizes that Lewis is likely going to turn the trip into A lesson. A moral. A life principle. A Way. The fact that the narrator thinks Lewis will turn the trip into a lesson and a moral suggests that Lewis has previously turned other things into lessons and morals. Indeed, the narrator goes on to say that Lewis liked . . . to take some extremely specialized and difficult form of sport . . . and evolve a personal approach to it which he could then expound. None of the narrators statements indicate that Lewis is sometimes whimsical, or given to erratic behavior. On the contrary, the narrator seems to know what to expect; this suggests that Lewis behavior is predictable.. Choice (B) is incorrect. Someone who is callous is hard-hearted and unemotional. In lines 39-40, the narrator recognizes that Lewis is likely going to turn the trip into A lesson. A moral. A life principle. A Way. The fact that the narrator thinks Lewis will turn the trip into a lesson and a moral suggests that Lewis has previously turned other things into lessons and morals. Indeed, the narrator goes on to say that Lewis liked . . . to take some extremely specialized and difficult form of sport . . . and evolve a personal approach to it which he could then expound. There is no direct connection between wanting to teach others and being hard-hearted, and none of the narrators statements indicate that Lewis is callous. Rather, the narrator suggests that Lewis sometimes makes moral observations and likes to teach othershe is sometimes didactic..

Choice (C) is incorrect. Someone who is remiss is negligent or careless in his or her duties. In lines 39-40, the narrator recognizes that Lewis is likely going to turn the trip into A lesson. A moral. A life principle. A Way. The fact that the narrator thinks Lewis will turn the trip into a lesson and a moral suggests that Lewis has previously turned other things into lessons and morals. Indeed, the narrator goes on to say that Lewis liked . . . to take some extremely specialized and difficult form of sport . . . and evolve a personal approach to it which he could then expound. None of the narrators statements indicate that Lewis is sometimes negligent or careless. On the contrary, the narrator describes Lewis as being careful and focused when it comes to planning the trip and taking on forms of sport.. Choice (E) is incorrect. Someone who is impetuous is impulsive and passionate. In lines 39-40, the narrator recognizes that Lewis is likely going to turn the trip into A lesson. A moral. A life principle. A Way. The fact that the narrator thinks Lewis will turn the trip into a lesson and a moral suggests that Lewis has previously turned other things into lessons and morals. Indeed, the narrator goes on to say that Lewis liked . . . to take some extremely specialized and difficult form of sport . . . and evolve a personal approach to it which he could then expound. Lewis seems to be passionate about certain thingsthe groups upcoming trip and his personal approaches to sportsbut the narrators statements indicate that Lewis is also very single-minded and focused. Therefore, the narrators statements do not suggest that Lewis is sometimes impetuous. 24 Explanation for Correct Answer C. Choice (C) is correct. In lines 52-53, the narrator lists several sports that have attracted Lewis interest: fly casting . . . archery and weight lifting and spelunking. The narrator also states that what Lewis mainly . . . like[s] is doing things in a way that is strictly his ownLewis devises a personal approach which he can then expound, or explain in detail. The fact that Lewis likes to develop his own personal approach to a sport suggests that he prefers sports that allow room for individual expression.. Choice (A) is incorrect. In lines 52-53, the narrator lists several sports that have attracted Lewis interest: fly casting . . . archery and weight lifting and spelunking. Each of these sportsdoes require specialized equipment; indeed, in spelunkingcave explorationthe lack of proper equipment can put the spelunkers life in danger. Therefore, it does not make sense to say that Lewis prefers sports that do not require special equipment. Further, the narrator notes that what Lewis mainly . . . like[s] is devising a personal approach which he can then expound, or explain in detail. This suggests that Lewis prefers sports that allow room for individual expression.. Choice (B) is incorrect. In lines 48-54, the narrator notes that Lewis particularly likes sports he [can] do by himself and lists several sports that have attracted Lewis interest: fly casting . . . archery and weight lifting and spelunking. Although each of the sports can be practiced competitively, they are not inherently competitive because competition is characterized by rivalry between two or more parties. Lewis seems to prefer sports he can

do alone, not sports that would require at least two people to compete against each other. Further, the narrator notes that what Lewis mainly . . . like[s] is devising a personal approach which he can then expound, or explain in detail. This suggests that Lewis prefers sports that allow room for individual expression.. Choice (D) is incorrect. In lines 52-53, the narrator lists several sports that have attracted Lewis interest: fly casting . . . archery and weight lifting and spelunking. Although weight lifting requires strength, all of the sports listed can be practiced by people who are not extremely strong. Additionally, the narrator notes that in all of these sports, Lewis developed complete mystiques. In this context, a mystique is a special esoteric skill essential to an activity. Therefore, it might be more accurate to suggest that Lewis prefers sports that do not require great strength, but that do require special skillsspecial skills that Lewis devises. This also suggests that Lewis prefers sports that allow room for individual expression.. Choice (E) is incorrect. In lines 52-53, the narrator lists several sports that have attracted Lewis interest: fly casting . . . archery and weight lifting and spelunking. Earlier in the paragraph, the narrator also mentions that Lewis likes extremely specialized and difficult form[s] of sport. Sports that are specialized and difficult likely require specific training and equipment; the sports Lewis prefers might in fact pose risks for beginners. Further, the narrator notes that what Lewis mainly . . . like[s] is devising a personal approach which he can then expound, or explain in detail. This suggests that Lewis prefers sports that allow room for individual expression. 25 Explanation for Correct Answer C. Choice (C) is correct. Dry humor is a kind of subtle or understated humor. In this passage, Bobbys statement is not a joke with an explicit punch line, but the reader also should not take the statement literally. That is, middle-class householders do not actually lie down till the feelinga desire such as Lewis desire to explore a wild areapasses. Bobbys remarks are subtle and humorous, stating one thing but meaning another.. Choice (A) is incorrect. In this passage, Bobbys remarks can be read as a critique of the middle class because they suggest that middle-class householders are unwilling to take risks, such as the risk of exploring a wild area. However, this critique is not explicit, or expressed in a way that leaves no question about its meaning. Because middle-class householders do not literally lie down till the feeling passes, the reader must read between the lines to interpret Bobbys statement. Even if Bobbys remarks contain a criticism of the middle class, this criticism is not explicit.. Choice (B) is incorrect. In this passage, Bobby does not portray the middle class in a completely positive way: he describes the people in this group as unwilling to take risks, such as the risk of exploring a wild area. But Bobbys remarks do not suggest that he intends to cause harm or pain to the middle-class householders or that he is veilingor covering upany such malice..

Choice (D) is incorrect. A confession is a disclosure of ones sins or an acknowledgement of guilt. Bobbys remarks about middle-class householders lying down till the feelinga desire such as Lewis desire to explore a wild areapasses do not contain any confessions or refer to guilt or sin in any way.. Choice (E) is incorrect. A factual observation is a truthful statement based on something one has observed or measured. Bobbys remarks, however, are not meant to be understood literallyhe does not mean that middle-class householders actually lie down till the feeling passes when they are struck by a desire such as Lewis desire to explore a wild area. Further, the passage gives no indication that Bobby is referring to his own observations of middle-class householders. Bobbys statements are meant to be subtly humorous. Section #6: View Explanations 1 Explanation for Correct Answer D. Choice (D) is correct. It avoids the lack of parallelism of the original by providing a verb (practicing) that is parallel with the preceding verbs (reading and relaxing).. Choice (A) involves a lack of parallelism. The verb phrase to be practicing is not parallel with the preceding verbs (reading and relaxing).. Choice (B) results in a lack of parallelism and unidiomatic phrasing. The phrase for practicing is not parallel with preceding verbs (reading and relaxing). In addition, the phrase to spend the night . . . for is unidiomatic.. Choice (C) results in a lack of parallelism and awkward phrasing. The verb to practice is not parallel with the preceding verbs (reading and relaxing). In addition, the conjunction more than is awkward in this context.. Choice (E) results in a lack of parallelism and improper verb tense. The verb phrase having practiced is not parallel with the preceding verbs (reading and relaxing). In addition, the present perfect participle having practiced illogically indicates an action that was completed before the action of the past-tense verb in the sentence (decided). 2 Explanation for Correct Answer E. Choice (E) is correct. It avoids the sentence-fragment error of the original by providing a main verb to carry out the action of the sentence..

Choice (A) involves a sentence fragment. There is no main verb to carry out the action of the sentence, only the modifying phrase who identified . . . malaria.. Choice (B) involves wordiness, redundancy, and improper verb tense, resulting in faulty sentence structure. The subject, Sir Ronald Ross, is named at the beginning of the sentence, making the pronoun he before the verb phrase (has identified) unnecessary. In addition, the present perfect verb phrase has identified illogically indicates Rosss identification of the Anopheles mosquito as the transmitter of human malaria continues to the present; the simple past-tense verb identified should be used instead.. Choice (C) involves awkward phrasing and improper verb tense, resulting is faulty sentence structure. The conjunction and may be used to join two verb phrases; however there is no verb phrase before the conjunction; there is only the modifying phrase winner of the 1902 Nobel Prize for Physiology or Medicine. In addition, the present perfect verb phrase has identified illogically indicates Rosss identification of the Anopheles mosquito as the transmitter of human malaria continues to the present; the simple past-tense verb identified should be used instead.. Choice (D) results in a sentence fragment and improper verb tense. There is no main verb to carry out the action of the sentence. In addition, the present-tense verb phrase is identifying illogically implies that Rosss identification of the Anopheles mosquito as the transmitter of human malaria is ongoing. 3 Explanation for Correct Answer C. Choice (C) is correct. It avoids the modification error of the original by providing a subject (we) that can logically be modified by the sentences opening phrase (Traveling through Yosemite).. Choice (A) involves illogical modification. It does not make sense to say that the scenery was Traveling through Yosemite.. Choice (B) results in illogical modification. It does not make sense to say that the waterfalls and granite peaks were Traveling through Yosemite.. Choice (D) results in unclear modification. It is not clear what being beautiful is meant to modify.. Choice (E) results in illogical modification. It does not make sense to say that what we photographed was Traveling through Yosemite. 4 Explanation for Correct Answer C.

Choice (C) is correct. It avoids the wordiness of the original by deleting the unnecessary phrase with his purpose being.. Choice (A) involves wordiness. The phrase with his purpose being is unnecessary and should be deleted.. Choice (B) results in wordiness and an inappropriate verb form. The phrase and his purpose was is unnecessary. In addition, the infinitive to recount should be used instead of the participle recounting.. Choice (D) involves an error in verb tense that results in faulty sentence structure. With the sentence rewritten this way, Firdawsis act of recounting is simultaneous with his act of composing, and thus the participle recounting should be used in place of the past-tense verb recounted. In addition, the phrase thus . . . kings should be set off with a comma.. Choice (E) results in a run-on sentence. Two independent clauses (The poet Firdawsi composed . . . in 1010 and he recounted . . . kings) are joined without punctuation. 5 Explanation for Correct Answer B. Choice (B) is correct. It avoids the comma splice of the original by replacing the second independent clause with the modifying phrase with works inspired by African American culture.. Choice (A) involves a comma splice. Two independent clauses (Judith Jamison has enriched the world of dance and she uses as her works inspiration African American culture) are joined by only a comma.. Choice (C) results in a sentence fragment. There is no main verb to carry out the action of the sentence, only the modifying phrase who has enriched . . . culture.. Choice (D) results in a sentence fragment. There is no main verb to carry out the action of the sentence, only a series of modifying phrases.. Choice (E) results in awkward, illogical phrasing. It does not make sense to say that works had the inspiration of African American culture. People can have inspiration, but works cannot. 6 Explanation for Correct Answer C. Choice (C) is correct. It avoids the faulty sentence structure of the original by providing a main verb (influenced) for the relative clause beginning with that (that family storytellers powerfully influenced . . .)..

Choice (A) involves faulty sentence structure. There is no main verb to carry out the action of the relative clause that her writing . . .. Choice (B) results in faulty sentence structure. There is no main verb to carry out the action of the relative clause that her writing . . .. Choice (D) results in faulty sentence structure. There is no main verb to carry out the action of the relative clause that storytellers in the family . . ., only the participle being.. Choice (E) results in subject-verb disagreement. The singular verb was does not agree with its plural subject, storytellers. 7 Explanation for Correct Answer C. Choice (C) is correct. It avoids the modification error of the original by providing a subject (Antonio) that can logically be modified by the phrase Finding the Baltimore waterfront fascinating.. Choice (A) involves unclear modification. The subject of the sentence, all that there was to see, cannot logically be modified by the phrase Finding the Baltimore waterfront fascinating.. Choice (B) results in a comma splice. Two independent clauses (Antonio found the Baltimore waterfront fascinating and he thoroughly explored all that there was to see) are joined by only a comma.. Choice (D) involves awkward phrasing. It does not make sense to say that The Baltimore waterfront . . . is why Antonio thoroughly explored all that there was to see.. Choice (E) involves unidiomatic phrasing. It is unidiomatic to say The Baltimore waterfront can be found fascinating. The verb found should be deleted. 8 Explanation for Correct Answer A. Choice (A) is correct. It avoids the verb-form errors of the other options by providing the present particle shaving to modify each.. Choice (B) results in an inappropriate verb form. The phrase modifying each requires a participle (shaving), not a main verb (had shaved).. Choice (C) results in an inappropriate verb form. The phrase modifying each requires a participle (shaving), not a main verb (was shaving)..

Choice (D) results in unidiomatic phrasing and an inappropriate verb form. It is not idiomatic to follow each with who. In addition, the phrase modifying each requires a participle (shaving), not a main verb (had been shaving).. Choice (E) results in unidiomatic phrasing and an inappropriate verb form. It is not idiomatic to follow each with who. In addition, the phrase modifying each requires a participle (shaving), not a main verb (shaved). 9 Explanation for Correct Answer D. Choice (D) is correct. It avoids the modification error of the original by placing the phrase eat large numbers of insects so that it clearly applies to bats.. Choice (A) involves unclear modification. The subject of the sentence, some people, cannot logically be modified by Because of their ability to eat large numbers of insects.. Choice (B) results in illogical phrasing. The structure of this sentence illogically indicates that some people possess the ability to eat large numbers of insects.. Choice (C) results in a vague pronoun. It is unclear whether the possessive pronoun their refers to bats or people.. Choice (E) results in a comma splice. Two independent clauses (Bats can eat large numbers of insects and because of this . . . backyards) are joined by only a comma. 10 Explanation for Correct Answer A. Choice (A) is correct. It avoids the errors of the other options by providing a concise, logical, and clear introductory phrase.. Choice (B) results in awkward phrasing and wordiness. The phrase the fact of is unnecessary and should be deleted.. Choice (C) results in inappropriate word choice. The introductory phrase Besides their having talked does not provide the appropriate contrast between the companies talking about ecology and their spending little to fight pollution.. Choice (D) results in inappropriate word choice. The introductory phrase In addition to their talking does not provide the appropriate contrast between the companies talking about ecology and their spending little to fight pollution.. Choice (E) results in awkward, confusing phrasing. It is not clear that the phrase one and the other refers to major companies.

11 Explanation for Correct Answer A. Choice (A) is correct. It avoids the errors of the other options by providing a phrase (the education of the public) that is parallel with the preceding phrase (the enforcement of waste-disposal regulations). In addition, the conjunction and appropriately follows the conjunction both.. Choice (B) results in faulty coordination and a lack of parallelism. The two parts of the correlative both . . . and . . . must be followed by grammatically similar elements. However, the second part of the correlative (and) is missing, and the verb phrase educating the public is not parallel with the preceding noun phrase (the enforcement of waste-disposal regulations).. Choice (C) results in a lack of parallelism. The two parts of the correlative both . . . and . . . must be followed by grammatically similar elements. However, the public being educated is both awkward and not parallel with the enforcement of waste-disposal regulations.. Choice (D) results in faulty coordination. The correct form of the correlative is both . . . and . . ., not both . . . along with . . .. Choice (E) results in faulty coordination. The correct form of the correlative is both . . . and . . ., not both . . . in combination with . . . 12 Explanation for Correct Answer B. The error in this sentence occurs at (B). The objective pronoun me is not appropriate in the subject of a sentence and should be replaced with the nominative pronoun I.. There is no error at (A). The participle Hearing appropriately introduces the participle phrase Hearing the unexpected loud noise, which modifies the subject (Cindy, Leroy, and I).. There is no error at (C). The plural verb phrase were . . . startled agrees with the plural subject (Cindy, Leroy, and I). In addition, the adverb so appropriately modifies the past participle startled.. There is no error at (D). The prepositional phrase out of our seats appropriately modifies the verb jumped.. There is an error in the sentence. 13 Explanation for Correct Answer D.

The error in this sentence occurs at (D). The present-tense verb writes inappropriately describes an action from the past (as indicated by the phrase early in her career) and should be replaced with the past-tense verb wrote.. There is no error at (A). The adjective detailed appropriately modifies the noun descriptions, and the preposition of idiomatically follows descriptions and introduces the prepositional phrase of nineteenth-century domestic life.. There is no error at (B). The phrase such as idiomatically indicates Little Women is an example of the noun novels.. There is no error at (C). The conjunction but appropriately indicates contrast between the phrase that precedes the comma (Many admire Louisa May Alcott for her detailed descriptions of nineteenth-century domestic life . . .) and the phrase that follows but (few have read the lurid thrillers . . .). In addition, the plural pronoun few agrees with the plural verb phrase have read.. There is an error in the sentence. 14 Explanation for Correct Answer C. The error in this sentence occurs at (C). As indicated by the past-tense verb saw, this sentence describes the action in a play in the historical past rather than in the narrative present. Thus the present progressive tense of the verb believing is inappropriate and should be replaced with the past-tense verb believed.. There is no error at (A). The reflexive verb himself appropriately refers to the noun character.. There is no error at (B). The prepositional phrase as the savior appropriately modifies the verb saw to describe how the character saw himself.. There is no error at (D). As indicated by the past-tense verb saw, this sentence describes the action in a play in the historical past rather than in the narrative present. Thus the pasttense verb phrase could do is used appropriately in the sentence.. There is an error in the sentence. 15 Explanation for Correct Answer B. The error in this sentence occurs at (B). The correlative conjunction neither, which links Leslie and Philip, is inappropriately paired with the conjunction or instead of the idiomatic nor..

There is no error at (A). The preposition Unlike appropriately establishes contrast between Thomas and Leslie (and Philip).. There is no error at (C). The article an appropriately modifies the noun interest, which is the correct word to describe a desire or inclination.. There is no error at (D). The noun career is the correct word to describe a profession. In addition, the preposition in idiomatically follows the noun career and appropriately introduces the prepositional phrase in law.. There is an error in the sentence. 16 Explanation for Correct Answer C. The error in this sentence occurs at (C). The present perfect tense verb phrase has been inappropriately describes an event that continues to the present and should be replaced with the past perfect tense (had been) or simple past tense (was) to indicate an action that was completed in the past.. There is no error at (A). The noun subject is an appropriate word to mean topic or theme, and the preposition of idiomatically follows subject and appropriately introduces the prepositional phrase of Felipe Alfaus second novel.. There is no error at (B). The comparative adverb more appropriately modifies the adjective 40. The conjunction than appropriately links the comparative adverb to the adjective it modifies.. There is no error at (D). The past participle written appropriately describes the subject novel.. There is an error in the sentence. 17 Explanation for Correct Answer D. The error in this sentence occurs at (D). The complement a slave laborer is a singular noun phrase and does not agree with the plural noun workers; it should be replaced with the plural noun phrase slave laborers.. There is no error at (A). The plural verb phrase have stopped buying agrees with the plural subject consumers..

There is no error at (B). The preposition in appropriately introduces the prepositional phrase in which workers are essentially . . . , and the pronoun which correctly refers to countries.. There is no error at (C). The adverb essentially appropriately modifies the verb are.. There is an error in the sentence. 18 Explanation for Correct Answer C. The error in this sentence occurs at (C). The plural pronoun they inappropriately refers to the singular noun tax and should be replaced with it.. There is no error at (A). The singular past progressive verb phrase was evaluating agrees with the singular subject mayor and appropriately describes an action that was in progress at a specific point in the past.. There is no error at (B). The future conditional verb phrase would generate appropriately describes an action that may happen in the future on the condition that another event happens.. There is no error at (D). The adverb disproportionately appropriately modifies the verb affect.. There is an error in the sentence. 19 Explanation for Correct Answer E. The correct choice is (E). This sentence contains no errors.. There is no error at (A). The adverb long in the verb phrase has long been regarded appropriately modifies the present perfect verb phrase, which appropriately describes an action that began in the past and continues to the present.. There is no error at (B). The preposition as appropriately introduces the prepositional phrase as a means, which modifies the verb phrase has long been regarded. Furthermore, the noun phrase a means is an appropriate phrase to mean method or way.. There is no error at (C). The singular present perfect tense verb phrase has shown agrees with the singular subject research and appropriately describes an action that began in the past and continues to the present.. There is no error at (D). The singular verb phrase does have agrees with the singular pronoun it.

20 Explanation for Correct Answer B. The error in this sentence occurs at (B). The singular verb holds does not agree with the plural subject managers and should be replaced with the plural verb hold.. There is no error at (A). The adverb fully appropriately modifies the complement staffed.. There is no error at (C). The demonstrative pronoun that appropriately modifies the possessive noun shifts, and this phrase describes the noun work.. There is no error at (D). The singular pronoun it appropriately refers to the singular noun work.. There is an error in the sentence. 21 Explanation for Correct Answer C. The error in this sentence occurs at (C). The phrase the combination of . . . and . . . should take parallel objects. The verb and adjective phrase being agile is not parallel with the noun strength and should be replaced with the noun agility.. There is no error at (A). The plural verb have agrees with the plural subject Members.. There is no error at (B). The past participle shown functions appropriately as part of the verb phrase have . . . shown. The conjunction how introduces the clause the combination . . . can produce beautiful movements.. There is no error at (D). The singular verb phrase can produce agrees with the singular subject combination.. There is an error in the sentence. 22 Explanation for Correct Answer D. The error in this sentence occurs at (D). The plural verb are does not agree with the singular subject obsession and should be replaced with the singular verb is.. There is no error at (A). The singular verb features agrees with the singular subject novel..

There is no error at (B). The possessive pronoun whose appropriately refers to the noun character.. There is no error at (C). The prepositional phrase with saving time and money is appropriately introduced by the preposition with, which idiomatically follows the noun obsession.. There is an error in the sentence. 23 Explanation for Correct Answer A. The error in this sentence occurs at (A). The preposition to does not idiomatically follow the past participle arrived and should be replaced with in.. There is no error at (B). The adverb moodily appropriately modifies the verb watches.. There is no error at (C). The singular verb watches agrees with the singular subject Nick Carraway.. There is no error at (D). The noun tip is used appropriately to describe a location, and the preposition of idiomatically follows the word tip to introduce the prepositional phrase of Long Island.. There is an error in the sentence. 24 Explanation for Correct Answer E. The correct choice is (E). This sentence contains no errors.. There is no error at (A). The preposition Despite appropriately introduces the prepositional phrase Despite the efforts of the publicity subcommittee, which modifies the verb attended.. There is no error at (B). The noun phrase hardly anyone idiomatically functions as the subject of the verb attended.. There is no error at (C). The past perfect tense verb phrase had been appropriately describes an action that began and ended in the past.. There is no error at (D). The adverbial phrase so painstakingly appropriately modifies the verb phrase had been planned.25 Explanation for Correct Answer C.

The error in this sentence occurs at (C). The singular verb continues does not agree with the plural subject flights and should be replaced with the plural verb continue.. There is no error at (A). The adverb seemingly appropriately modifies the adjective effortless, which modifies the noun flights.. There is no error at (B). The past participle achieved appropriately modifies the noun flights, and the preposition through idiomatically follows achieved and appropriately introduces the prepositional phrase through the use.. There is no error at (D). The relative pronoun who appropriately introduces the relative clause who see the play, which modifies the pronoun those.. There is an error in the sentence. 26 Explanation for Correct Answer D. The error in this sentence occurs at (D). The preposition for does not idiomatically follow the noun offers in this context and should be replaced with the preposition of. (These were offers to provide assistance, not offers of terms for receiving assistance.). There is no error at (A). The preposition to appropriately introduces the prepositional phrase to intervene, which modifies the adjective prepared. The preposition in idiomatically follows the verb intervene and appropriately introduces the prepositional phrase in the labor dispute.. There is no error at (B). The conjunctional phrase even though is appropriately used to indicate contrast between two ideas (Mediators were standing by and both sides had refused earlier offers . . .).. There is no error at (C). The past perfect tense verb phrase had refused is appropriately used to describe an action that began and ended in the past.. There is an error in the sentence. 27 Explanation for Correct Answer E. The correct choice is (E). This sentence contains no errors.. There is no error at (A). The indefinite pronoun what is used appropriately to refer to an implied antecedent.. There is no error at (B). The adjective any appropriately modifies the noun bird..

There is no error at (C). The preposition with appropriately introduces the prepositional phrase with even the slightest variation, and the adverb even appropriately modifies the adjective slightest.. There is no error at (D). The article the and the superlative slightest appropriately modify the noun variation. 28 Explanation for Correct Answer C. The error in the sentence occurs at (C). The possessive pronoun her is vague and does not indicate whether Sam is the son of Ms. Perez or of Ms. Tanaka; it should be replaced with the appropriate proper noun.. There is no error at (A). The correlative conjunction Neither is appropriately followed by nor.. There is no error at (B). The singular verb believes is appropriate. Although there are two subjects (Ms. Perez and Ms. Tanaka), they are separated by the conjunction nor; therefore, the case is determined by the subject closest to the verb, which is the singular Ms. Tanaka.. There is no error at (D). The future tense of the verb phrase will lead is appropriate to describe the possible results of Sams watching too much television.. There is an error in the sentence. 29 Explanation for Correct Answer E. The correct choice is (E). This sentence contains no errors.. There is no error at (A). The reflexive pronoun herself correctly refers to the noun accountant.. There is no error at (B). The infinitive verb phrase to help appropriately functions as the direct object of the verb offered.. There is no error at (C). The adverb as appropriately modifies the participle complicated. In addition, the pronoun they appropriately refers to the plural noun accounts and agrees with the plural past-tense verb were.. There is no error at (D). The preposition by idiomatically follows the participle complicated (complicated . . . by) and appropriately introduces the prepositional phrase by his unusual system.

30 Explanation for Correct Answer E. Choice (E) is correct. This version of the sentence creates a dependent clause after the comma, making the sentence grammatically correct. In addition, it is clear that the demonstrative pronoun those refers to purists.. Choice (A) is unsatisfactory. Two independent clauses (A recent version . . . purists and they are people . . . exactly) are joined by only a comma, creating a comma splice.. Choice (B) is unsatisfactory. While a semicolon can join two independent clauses, the clauses should be of equal importance. In this case, the definition of purists is subordinate to the main point of the sentence (that a recent film version of Romeo and Juliet drew harsh reviews), so joining the two clauses with a semicolon is inappropriate. Furthermore, since the second clause gives a definition of the word purists, the habitual/universal present tense should be used, not the past tense.. Choice (C) is unsatisfactory. Revising the sentence in this way would illogically imply that the recent film version of Romeo and Juliet (not the purists) was expecting filmmakers to follow the text exactly.. Choice (D) is unsatisfactory. It is unclear to what (reviews or purists) the demonstrative pronoun These refers. 31 Explanation for Correct Answer C. Choice is (C) is correct. Replacing ones with reviews clarifies that the topic of the sentence is positive reviews.. Choice (A) is unsatisfactory. Replacing ones with scenes creates an illogical sentence. Scenes in a film version of Romeo and Juliet cannot have expressed relief that Shakespeare was not around feel the insult.. Choice (B) is unsatisfactory. Replacing ones with instances replaces one vague word with another vague word. Sentence 3 requires a more specific word to indicate who or what expressed relief.. Choice (D) is unsatisfactory. Replacing ones with remakes creates an illogical sentence. Remakes of Romeo and Juliet cannot have expressed relief that Shakespeare was not around feel the insult.. Choice (E) is unsatisfactory. Replacing ones with sections does not provide clarity. Sections might refer to parts of the film or parts of the reviews. 32

Explanation for Correct Answer C. Choice (C) is correct. This version of sentence 5 creates a complete sentence. In addition, the transitional phrase (Another supposed outrage) makes clear the relationship between sentences 4 and 5.. Choice (A) is unsatisfactory. This version of sentence 5 is a fragment with no verb to carry out the action of the sentence. Furthermore, the lack of a phrase indicating a transition from discussing Shakespeares Romeo and Juliet to Jane Austens Emma leaves the essay illogically considering Shakespeares possible reaction to a movie of Austens novel.. Choice (B) is unsatisfactory. While this version creates a complete sentence, there is no word or phrase to provide a transition between the distinct ideas in sentences 4 and 5.. Choice (D) is unsatisfactory. While this version of sentence 5 creates a complete sentence and provides a transition between the ideas in sentences 4 and 5, the meaning of the sentence becomes contradictory to the rest of the draft. In the second paragraph, the writer states, I see nothing wrong with creative remakes, so the writer would not logically take offense to a remake of Jane Austens novel.. Choice (E) is unsatisfactory. This version of sentence 5 creates a complete sentence and provides a transition from sentence 4; however, the transition Yet implies that the filmClueless will be an example contrary to the topic of sentence 4. In fact, Clueless is an example similar to the film discussed in sentence 4. Hence Yet is not an appropriate transition between the ideas in sentences 4 and 5. 33 Explanation for Correct Answer A. Choice (A) is correct. In the first paragraph, the writer cites criticism of modern remakes of Shakespeares plays and asks questions such as, Wouldnt he be horrified to see his play open with a gang shoot-out at a gas station? The second paragraph illuminates the writers personal view by stating, I see nothing wrong with creative remakes. In other words, the writer uses the strategy of elaborating on a view in the first paragraph as a way of introducing his or her own contrasting argument.. Choice (B) is unsatisfactory. While some modern remakes are described in detail in the first paragraph, there is no mention of a personal experience.. Choice (C) is unsatisfactory. The first paragraph mentions film reviews, but there is no real analysis of these reviews or of modern criticism in general.. Choice (D) is unsatisfactory. The first paragraph describes modern film versions of classic works by Shakespeare and Austen. There is no mention of a new, unconventional approach to fiction writing..

Choice (E) is unsatisfactory. The first paragraph mentions that the only positive reviews for one film from one type of reviewer (purists) were those that expressed relief that Shakespeare was not around to feel the insult. While this could be considered playful, there is no reason to assume that much of film criticism is this way. 34 Explanation for Correct Answer E. Choice (E) is correct. This sentence links the two parts of sentence 11 with the conjunction as, so that the phrase distant relatives of his own warring characters clearly refers to the swaggering teenagers in the movie. In addition, the sentence is grammatically correct.. Choice (A) is unsatisfactory. The original version of sentence 11 joins two independent clauses (No doubt he . . . movie and they would be . . . characters) with only a comma, creating a comma splice. In addition, the future conditional tense of the verb would be is illogical in this sentence.. Choice (B) is unsatisfactory. This version of sentence 11 joins two independent clauses (No doubt he . . . movie and they are . . . characters) with only a comma and so does not correct the comma splice of the original.. Choice (C) is unsatisfactory. While a semicolon can be used to join two independent clauses of equal value, the past tense of the verb were is illogical in this sentence. It does not make sense that the characters in a modern film would at one point in the past be distant relatives of Shakespeares characters.. Choice (D) is unsatisfactory. This revision changes the meaning of sentence 11. The preposition for can idiomatically follow the verb recognize in other contexts. Revising the sentence this way illogically implies that Shakespeare would formally acknowledge (not perceive as someone previously known) the modern film characters for being distant relatives of his own characters. 35 Explanation for Correct Answer B. Choice (B) is correct. The conditional future tense of the verb phrase would see is appropriate, because the action could only take place if Austen were alive. In addition, the adverb too provides a logical transition from the idea in sentence 11, linking Shakespeare and Austen.. Choice (A) is unsatisfactory. Revising sentence 12 this way implies that at some point in the past Austen could have seen aspects of her characters in the modern characters ofClueless. Given the vastly different time periods of the authors life and the modern film, this revision creates an illogical sentence..

Choice (C) is unsatisfactory. Revising sentence 12 in this way implies that at some point in the past Austen would have been able to see aspects of her characters in the modern characters of Clueless. Given the vastly different time periods of the authors life and the modern film, this revision creates an illogical sentence. In addition, the transition However inappropriately implies that sentence 12 contains an idea contradictory to the idea in sentence 11.. Choice (D) is unsatisfactory. While the future conditional verb tense of would see makes sense, beginning sentence 12 with the transitional phrase In addition to this creates an imprecise transition between sentences 11 and 12. Starting sentence 12 with In addition to this illogically implies that Austens seeing traces of her characters in Clueless logically flows from or is associated with Shakespeares recognizing his characters in a modern version of Romeo and Juliet.. Choice (E) is unsatisfactory. While the transition Likewise creates a logical link between sentence 11 and sentence 12, the present tense of the verb phrase can see is illogical considering Austen (who published Emma in 1815) is obviously no longer alive. Section #7: View Explanations 1 Explanation for Correct Answer D. Choice (D) is correct. "Eliminated" means gotten rid of or removed from consideration. The sentence discusses the possibility that a politician guilty of "corrupt dealings" might be reelected. A politician who has earned . . . disgrace, or has lost honor, is extremely unlikely to be reelected, so it makes sense to say that the "possibility of his being reelected" was eliminated.. Choice (A) is incorrect. "Ensured" means guaranteed or made certain. The sentence discusses the possibility that a politician guilty of "corrupt dealings" might be reelected. It does not make sense to say that the possibility of a politician's reelection was ensured, or made certain, as a result of the man's "disgrace," or loss of honor. Any chance such a politician once had for reelection would most likely be eliminated, not ensured.. Choice (B) is incorrect. "Approved" means accepted or endorsed. The sentence discusses the possibility that a politician guilty of "corrupt dealings" might be reelected. It does not make sense to say that the possibility of a politician's reelection was approved as a result of the man's "disgrace," or loss of honor. Any chance such a politician once had for reelection would most likely be eliminated, not approved.. Choice (C) is incorrect. "Belittled" means criticized or disparaged. The sentence discusses the possibility that a politician guilty of "corrupt dealings" might be reelected. It does not make sense to say that the possibility of a politician's reelection was belittled, or criticized,

as a result of the man's "disgrace," or loss of honor; the politician would surely be criticized, but not the possibility of his reelection. Any chance such a politician once had for reelection would most likely be eliminated, not belittled.. Choice (E) is incorrect. "Defended" means guarded or protected. The sentence discusses the possibility that a politician guilty of "corrupt dealings" might be reelected. It does not make sense to say that the possibility of a politician's reelection was defended, or protected, as a result of the man's "disgrace," or loss of honor. Any chance such a politician once had for reelection would most likely be eliminated, not defended. 2 Explanation for Correct Answer C. Choice (C) is correct. In this context, "selective" means choosy: selective editors would only include the very best submissions. "Inclusive" means the opposite of selective: inclusive editors would go out of their way to include many different submissions of varying quality. The fact that the material in the anthology was of "uneven," or varying, quality, indicates that the editors were inclusive. Further, the structure of the sentence ("Although . . .) suggests that the editors turned out to be the opposite of what they were reputed to be. The opposite of being inclusive is being selective.. Choice (A) is incorrect. "Amateurish" means lacking mastery. An amateurish performance is a performance by someone who has not mastered the skill required, and could be described as varying in quality. In this context, "professional" means roughly the opposite of amateurish: masterful. The structure of the sentence (Although . . .) suggests that the editors turned out to be the opposite of what they were reputed to be, but the fact that the material the editors included in the anthology was of "uneven," or varying, quality, suggests that the editors were as amateurish as they were reputed to be. Further, including works of varying quality does not suggest that the editors were too professional, or too masterful.. Choice (B) is incorrect. "Lax" means overly tolerant or permissive. "Harsh" means overly restrictive or judgmental. The structure of the sentence (Although . . . ) suggests that the editors turned out to be the opposite of what they were reputed to be, but the fact that the material the editors included in the anthology was of "uneven," or varying, quality, suggests that the editors were as lax as they were reputed to be. Further, the inclusion of works of varying quality suggests that rather than being too judgmental, the editors were not harsh enough.. Choice (D) is incorrect. "Judgmental" means judging harshly. "Discriminating" means distinguishing one thing from another, or, in this context, distinguishing submissions of the highest quality from all others. The fact that the material the editors included in the anthology was of "uneven," or varying, quality, suggests that the editors were not particularly discriminating, so it does not make sense to suggest that they were too discriminating. Further, the structure of the sentence (Although . . .) suggests

that the editors turned out to be the opposite of what they were reputed to be. Being judgmental is not the opposite of being too discriminating.. Choice (E) is incorrect. In this context, "sensitive" means aware of the feelings and attitudes of others. "Insightful" means perceptive or able to see the inner nature of things. The structure of the sentence (Although . . .) suggests that the editors turned out to be the opposite of what they were reputed to be. In this context, the terms sensitive and insightful mean essentially the same thing: perceptive. So these terms are not opposite in meaning. Further, the fact that the material the editors included in the anthology was of "uneven," or varying, quality, suggests that the editors were not particularly perceptive. 3 Explanation for Correct Answer D. Choice (D) is correct. "Succinct" means concise, with no wasted words. "Enlightening" means illuminating or instructive. The structure of the sentence indicates that the words after the colon essentially restate what has come before. The terms succinct and enlightening best complete the sentence: a presentation that was both succinct and enlightening would accurately be described as brief and instructive.. Choice (A) is incorrect. "Verbose" means wordy. "Mundane" means commonplace. The structure of the sentence indicates that the words after the colon essentially restate what has come before. A presentation that was both verbose and mundane would never be described as brief and instructive; a brief presentation is the opposite of a verbose one, and a mundane presentation might not be particularly instructive.. Choice (B) is incorrect. "Concise" means short and to the point. "Elaborate" means complex and detailed. The structure of the sentence indicates that the words after the colon essentially restate what has come before. A concise presentation might be accurately described as brief and instructive, but an elaborate one would not. An elaborate presentation is the opposite of a brief one.. Choice (C) is incorrect. "Comprehensive" means covering all aspects of a subject. "Edifying" means instructive. The structure of the sentence indicates that the words after the colon essentially restate what has come before. While the term edifying logically fits in the second blank, comprehensive does not fit in the first blank. A comprehensive presentation would not likely be brief.. Choice (E) is incorrect. "Provocative" means stimulating or controversial. Technical" means specialized and detailed. The structure of the sentence indicates that the words after the colon essentially restate what has come before. A presentation that is provocative and technical is not necessarily brief or instructive, so neither term is a good fit. 4 Explanation for Correct Answer E.

Choice (E) is correct. "Preeminence" is the quality of having superior rank or importance. The sentence indicates that the magazine has a "large circulation" (in this context, the term "circulation" refers to sales) and has enjoyed a certain position that has only recently been challenged by other magazines. This enjoyable position, characterized by high sales and few challengers or competitors, is aptly described by the term "preeminence.". Choice (A) is incorrect. An aggregation" is a collection made up of distinct parts. The sentence indicates that the magazine has a "large circulation" (in this context, the term "circulation" refers to sales) and has enjoyed a certain position that has only recently been challenged by other magazines. This enjoyable position, characterized by high sales and few challengers or competitors, would not likely be described as an aggregation.. Choice (B) is incorrect. An inclination" is a tendency or slant. The sentence indicates that the magazine has a "large circulation" (in this context, the term "circulation" refers to sales) and has enjoyed a certain position that has only recently been challenged by other magazines. This enjoyable position, characterized by high sales and few challengers or competitors, would not likely be described as an inclination.. Choice (C) is incorrect. A prognosis" is a forecast or prediction, and typically refers to a patient's recovery from an illness or disease. The sentence indicates that the magazine has a "large circulation" (in this context, the term "circulation" refers to sales) and has enjoyed a certain position that has only recently been challenged by other magazines. This enjoyable position, characterized by high sales and few challengers or competitors, would not logically be described as a prognosis.. Choice (D) is incorrect. "Retrenchment" means cutting back on expenses. The sentence indicates that the magazine has a "large circulation" (in this context, the term "circulation" refers to sales) and has enjoyed a certain position that has only recently been challenged by other magazines. While the magazine might respond to the challenge by retrenching, or trying to save money, this enjoyable position, characterized by high sales and few challengers or competitors, would not logically be described as a retrenchment. 5 Explanation for Correct Answer A. Choice (A) is correct. "Unequivocal" means clear and without a doubt. The structure of the sentence indicates that the words after the colon essentially restate what has come before. Because unequivocal opinions are opinions that leave no doubt about a person's intentions, the term "unequivocal" best completes the sentence.. Choice (B) is incorrect. "Effusive" means enthusiastic. Although a judge's opinions might be described as enthusiastic, the term effusive does not logically complete the sentence. The structure of the sentence indicates that the words after the colon essentially restate what has come before, and there is no connection between being enthusiastic and leaving no doubt of one's intentions..

Choice (C) is incorrect. "Incorrigible" means stubborn or unruly. The structure of the sentence indicates that the words after the colon essentially restate what has come before. There is no connection between being unruly and leaving no doubt of one's intentions, so the term "incorrigible" does not logically complete the sentence.. Choice (D) is incorrect. "Tenuous" means thin or shaky. Although a judge's opinions might be described as shaky, the term tenuous does not logically fit the blank. The structure of the sentence indicates that the words after the colon essentially restate what has come before, and there is no connection between being tenuous and leaving no doubt of one's intentions.. Choice (E) is incorrect. "Ineffable" means indescribable. It does not make sense to characterize a judge's published opinions as indescribable. Further, the structure of the sentence indicates that the words after the colon essentially restate what has come before, and there is no connection between being ineffable and leaving no doubt of one's intentions. 6 Explanation for Correct Answer D. Choice (D) is correct. In the first sentence the author introduces the subject of movements. He or she defines the term movement, explaining that a movement is a continuous, collective effort to bring about fundamental social reform.. Choice (A) is incorrect. In the first sentence the author introduces the subject of movements and defines a movement as a continuous, collective effort to bring about fundamental social reform. The author does not discuss any opinions, controversial or otherwise, anywhere in the passage.. Choice (B) is incorrect. In the first sentence the author introduces the subject of movements and defines a movement as a continuous, collective effort to bring about fundamental social reform. Later in the passage the author does discuss the objective of the Black freedom movement, but he or she does not at any time address the effectiveness of the movement.. Choice (C) is incorrect. In the first sentence the author introduces the subject of movements and states that a movement is a continuous, collective effort to bring about fundamental social reform. Rather than providing an example of an abstract idea, the author is defining the meaning of a term: movement.. Choice (E) is incorrect. In the first sentence the author introduces the subject of movements and states that a movement is a continuous, collective effort to bring about fundamental social reform. Rather than offering a solution to a problem, the author is defining the meaning of a term: movement. 7

Explanation for Correct Answer B. Choice (B) is correct. In lines 7-9 the author states that the objective of the Black freedom struggle was to transform, or change, the way Black Americans were viewed and treated. In lines 9-12 the author suggests that Black writers and artists were vital to the movement because they sought to change the way Black Americans were portrayed, or depicted, in literature and the arts. The way a group is depicted in the arts would likely have a strong effect on the way that group is viewed and treated.. Choice (A) is incorrect. The author states in lines 7-12 that Black writers and artists were vital to the freedom struggle, which hoped to change the way Black Americans were viewed and treated, because they sought to change the way Black Americans were portrayed in literature and the arts. Rather than promoting their own artistic freedom, these writers and artists wanted to change the way others depicted Black Americans in the arts.. Choice (C) is incorrect. The artists of the Black freedom movement may have produced protest art that documented the Black struggle, but the author does not indicate this. Instead, the author states in lines 7-12 that Black writers and artists were vital to the freedom struggle, which hoped to change the way Black Americans were viewed and treated, because they sought to change the way Black Americans were portrayed in literature and the arts.. Choice (D) is incorrect. The author does state that a movement is collaborative rather than . . . individualistic, but he or she does not suggest that Black writers and artists importance to the Black freedom movement was due to their opposition to excessive or extreme individualism. Rather, the author suggests that they were vital to the movement, which hoped to change the way Black Americans were viewed and treated, because they sought to change the way Black Americans were portrayed in literature and the arts.. Choice (E) is incorrect. The author states in lines 7-12 that Black writers and artists were vital to the freedom struggle, which hoped to change the way Black Americans were viewed and treated, because they sought to change the way Black Americans were portrayed in literature and the arts. These writers and artists may have followed a certain plan, but the author does not discuss a course of action or discuss its importance to the struggle; instead, the author emphasizes the importance of changing the way Black Americans were depicted in the arts. 8 Explanation for Correct Answer C. Choice (C) is correct. The passage explains how a slang word, cool, has remained popular far longer than other slang words. The author tells us that other words have not had the staying power of coolmany other slang words have been consigned to the scrap heap and are no longer used, but cool has stay[ed] alive..

Choice (A) is incorrect. The author does not mention any question and does not indicate that he or she is responding to a question. Instead, the author simply discusses the durability of the slang word cool.. Choice (B) is incorrect. The author does explain that slang words like groovy and farout are similar in meaning to the term cool, but the primary purpose of the passage is not to define an unusual expression; the expressions mentioned in the passage are likely familiar to many, and no real definitions are given. Rather, the main purpose of the passage is to note the durability of the slang word cool.. Choice (D) is incorrect. The author does not take a stance on the use of language. He or she mentions a variety of words that have been popular (hep, rad, tubular) but does not oppose the use of such slang. Instead, the author simply discusses the durability of the slang word cool.. Choice (E) is incorrect. The author does not discuss linguistic theory in this passage, and he or she does not challenge any theory. Instead, the author simply discusses the durability of the slang word cool. 9 Explanation for Correct Answer B. Choice (B) is correct. Ephemeral means lasting a very short time. In making the point that slang words are rarely in for long, the author compares slang words to fashion designs. Like slang words, says the author, a fashion trend usually has a short life expectancy in popular culture, so it is an example of something ephemeral.. Choice (A) is incorrect. Provocative means tending to provoke or excite. Slang words and fashion trends can sometimes cause excitement, but in the passage the author refers to fashion designs as an example of something that lasts a short time, not something provocative. The author makes the point that like fashion trends, slang words are rarely in for longthey have a short life expectancy in popular culture.. Choice (C) is incorrect. Pretentious means expressive of exaggerated importance. In the passage, the author refers to fashion designs as an example of something that lasts a short time. The author makes the point that like fashion trends, slang words are rarely in for longthey have a short life expectancy in popular culture. Nothing in the passage suggests that the author considers fashion designs pretentious or that he or she would compare slang words to something pretentious.. Choice (D) is incorrect. Esoteric means of special, rare, or unusual interest. In the passage, the author refers to fashion designs as an example of something that lasts a short time. The author makes the point that like fashion trends, slang words are rarely in for longthey have a short life expectancy in popular culture. Nothing in the passage suggests that the author considers fashion designs rare or unusual or that he or she would compare slang words to something esoteric..

Choice (E) is incorrect. Exotic means strikingly unusual. Fashion trends may sometimes be considered unusual, but in the passage the author refers to fashion designs as an example of something that lasts a short time, not something provocative. The author makes the point that like fashion trends, slang words are rarely in for longthey have a short life expectancy in popular culture. 10 Explanation for Correct Answer E. Choice (E) is correct. The first paragraph of the passage presents an astronomical enigma, or mystery: our nearest planetary neighbor, Venus, is so covered in clouds that it has been difficult to obtain accurate information about the planet. The following paragraphs trace various attempts to learn more about, and to explain, the "mystery" of Venus.. Choice (A) is incorrect. At no point does the passage criticize the lack of research on Venus, a topic of mystery to astronomers. In fact, the passage enthusiastically discusses different attempts to obtain accurate information about the planet and offers theories to explain why Venus is so different from Earth.. Choice (B) is incorrect. The author does present some speculative ideas about life on Venus (likely to be a very torrid sort of world, the surface might be largely ocean covered, There would be marshes, luxuriant vegetation . . . and primitive life-forms). But these ideas are part of a discussion of past attempts to understand Venus; for the most part, the author offers factual information rather than speculation. The primary purpose of the passage is to give a history of attempts to learn more about Venus, followed by a summation of what we now know to be true.. Choice (C) is incorrect. The author of the passage mentions, particularly in the third paragraph, some theories about Venus that have proven not to be valid. He or she even finds one of the theories "intriguing." But nothing in the passage suggests that the author "laments," or feels bad about, the fact that these theories did not accurately describe conditions on Venus's surface. The author is primarily reporting on the various attempts to learn more about, and to explain, the "mystery" of Venus.. Choice (D) is incorrect. The passage discusses particular attempts that have been made to learn more about, and to explain, the mystery of Venus. It also presents theories and explanations specific to conditions on that planet. There is no evidence that the passage was written to illustrate general principles of planetary research; rather, it seems to have been written to present information about a specific planet, Venus. 11 Explanation for Correct Answer A. Choice (A) is correct. A plausible supposition is a reasonable assumption or hypothesis. The statement in lines 11-14 presents a reasonable hypothesisbecause Venus's size and

gravitational field are similar to Earth's, Venus and Earth probably have similar atmospheres as welland then dismisses it: "this is emphatically not so.". Choice (B) is incorrect. The statement in lines 11-14 does reject a claim, explaining that the claim is wrong (this is emphatically not so). But the claimthat because Venus's size and gravitational field are similar to Earth's, Venus and Earth probably have similar atmospheres as wellcannot be described as outrageous. In fact, the author notes that, "logically [Venus and Earth] might be expected to have the same kind of atmosphere.". Choice (C) is incorrect. "Bolster" means support. The statement in lines 11-14 is part of a discussion of past attempts to learn more about Venus. The statement indicates that a logical assumptionthat Venus and Earth have similar atmospheresproved to be "emphatically not so." This statement is intended to show that a plausible assumption turned out to be false; it is not intended to support an accepted opinion.. Choice (D) is incorrect. The statement in lines 11-14 is part of a discussion of past attempts to learn more about Venus. The statement indicates that a logical assumptionthat Venus and Earth have similar atmospheresproved to be "emphatically not so." This statement is intended to show that a plausible assumption turned out to be false; it is not intended to summarize a particular experiment.. Choice (E) is incorrect. The statement in lines 11-14 does reject a claim, explaining that the claim is wrong ("this is emphatically not so"). But the claimthat because Venus's size and gravitational field are similar to Earth's, Venus and Earth probably have similar atmospheres as wellcannot be described as "a controversial hypothesis." In fact, the author notes that, "logically [Venus and Earth] might be expected to have the same kind of atmosphere." There is no indication that this assumption is or was controversial in any way. 12 Explanation for Correct Answer E. Choice (E) is correct. The passage traces various attempts to learn more about, and to explain, conditions on Venus. The third paragraph describes two early theories regarding Venus: "According to one theory, the clouds contained a great deal of water. . . . Another intriguing theory made Venus very similar to the Earth of over 200 million years ago." In 1962, the "first reliable information" was gathered about Venus, and these theories were proven to be invalid. The author includes these two theories as part of the history of scientists' attempts to understand Venus.. Choice (A) is incorrect. The passage traces various attempts to learn more about, and to explain, conditions on Venus. The third paragraph describes two early theories regarding Venus. The paragraph does not provide any evidence in support of these theories; in fact, it is clear that no evidence existed. In 1962, when the "first reliable information" was gathered about Venus, these theories were proven to be invalid..

Choice (B) is incorrect. The third paragraph describes two early theories regarding Venus: "According to one theory, the clouds contained a great deal of water. . . . Another intriguing theory made Venus very similar to the Earth of over 200 million years ago." However, there is no indication that these theories, which were later proven to be invalid, were ever popularnor are they challenged in the paragraph. The author is merely tracing the history of scientists' attempts to learn about, and to explain, conditions on Venus.. Choice (C) is incorrect. The third paragraph describes two early theories regarding Venus: "According to one theory, the clouds contained a great deal of water. . . . Another intriguing theory made Venus very similar to the Earth of over 200 million years ago." The paragraph gives equal weight to each hypothesis, so it makes no sense to say that the paragraph is focused on a particular hypothesis. Moreover, neither theory is shown to be misguided until the next paragraph, when the author discusses the "reliable information" that proved these theories to be invalid.. Choice (D) is incorrect. The third paragraph describes two early theories regarding Venus: "According to one theory, the clouds contained a great deal of water. . . . Another intriguing theory made Venus very similar to the Earth of over 200 million years ago." While these theories seem far-fetched today, in light of the "reliable information" we now possess, there is no indication in the paragraph that Venus has been romanticized, or idealized, throughout history. 13 Explanation for Correct Answer B. Choice (B) is correct. The reader can infer from the passage that "luxuriant vegetation . . . and primitive life-forms such as giant dragonflies" could not survive in an atmosphere of pure carbon dioxide. The third paragraph explains that, despite preliminary information suggesting that Venus's atmosphere consisted largely of carbon dioxide, "opinions differed," and some thought that Venus "might be largely ocean covered" or contain "primitive life-forms." But the fourth paragraph resolves the issue: information gathered by the American probe Mariner 2 revealed that "the atmosphere really is almost pure carbon dioxide. . . . All ideas of a pleasant, oceanic Venus had to be abandoned." Clearly, oceans and life-forms are incompatible with an atmosphere of pure, or almost pure, carbon dioxide.. Choice (A) is incorrect. The reader can infer from the passage that "luxuriant vegetation . . . and primitive life-forms such as giant dragonflies" could not survive in an atmosphere of pure carbon dioxide. The fourth paragraph resolves the issue: information gathered by the American probe Mariner 2 revealed that "the atmosphere really is almost pure carbon dioxide. . . . All ideas of a pleasant, oceanic Venus had to be abandoned." There is no indication that a warm and humid environment would prevent the growth of vegetation and other life-forms.. Choice (C) is incorrect. The reader can infer from the passage that "luxuriant vegetation . . . and primitive life-forms such as giant dragonflies" could not survive in an atmosphere of

pure carbon dioxide. The fourth paragraph resolves the issue: information gathered by the American probe Mariner 2 revealed that "the atmosphere really is almost pure carbon dioxide. . . . All ideas of a pleasant, oceanic Venus had to be abandoned." The last sentence of the third paragraph suggests that Venus might follow Earth's pattern of evolution, but the idea of evolution is not central to the hypothesis advanced in lines 28-30.. Choice (D) is incorrect. The reader can infer from the passage that "luxuriant vegetation . . . and primitive life-forms such as giant dragonflies" could not survive in an atmosphere of pure carbon dioxide. The fourth paragraph resolves the issue: information gathered by the American probe Mariner 2 revealed that "the atmosphere really is almost pure carbon dioxide. . . . All ideas of a pleasant, oceanic Venus had to be abandoned." There is no indication in the passage that photosynthesis is a relevant issue.. Choice (E) is incorrect. The reader can infer from the passage that "luxuriant vegetation . . . and primitive life-forms such as giant dragonflies" could not survive in an atmosphere of pure carbon dioxide. The fourth paragraph resolves the issue: information gathered by the American probe Mariner 2 revealed that "the atmosphere really is almost pure carbon dioxide. . . . All ideas of a pleasant, oceanic Venus had to be abandoned." There is no indication in the passage that creatures' ability to fly is relevant. 14 Explanation for Correct Answer B. Choice (B) is correct. According to lines 32-34, "the American probe Mariner 2 . . . gave us our first reliable information" about conditions on Venus. Clearly, the author believes that earlier evidence was unreliable, or untrustworthy.. Choice (A) is incorrect. According to lines 32-34, "the American probe Mariner 2 . . . gave us our first reliable information" about conditions on Venus. Clearly, the author believes that earlier evidence was unreliable, or untrustworthy. There is no indication that the quality of the data surprised the scientists.. Choice (C) is incorrect. According to lines 32-34, "the American probe Mariner 2 . . . gave us our first reliable information" about conditions on Venus. Clearly, the author believes that earlier evidence was unreliable, or untrustworthy. There is no indication of any records having been lost.. Choice (D) is incorrect. According to lines 32-34, "the American probe Mariner 2 . . . gave us our first reliable information" about conditions on Venus. There is no indication that scientists formulated any new theories as a result. Rather, the "surface proved to be very hot," supporting scientists' earlier contention that "Venus was likely to be a very torrid sort of world.". Choice (E) is incorrect. According to lines 32-34, "the American probe Mariner 2 . . . gave us our first reliable information" about conditions on Venus. The "surface proved to be very hot," supporting scientists' earlier contention that "Venus was likely to be

a very torrid sort of world." There is no suggestion that the theory confirmed by the data was obscure or implausible. 15 Explanation for Correct Answer D. Choice (D) is correct. Decisive means without doubt. The statement in lines 43-44 decisively answers the question posed at the beginning of the paragraph, "Why is Venus so unlike Earth?" The authors response, beginning with the words, "The answer can only lie . . ." indicates that there is only one possible answer to the question: Venus is unlike Earth because it is significantly closer to the Sun.. Choice (A) is incorrect. The statement in lines 43-44 decisively, or without doubt, answers the question posed at the beginning of the paragraph, "Why is Venus so unlike Earth?" The authors response, beginning with the words, "The answer can only lie . . ." indicates that there is only one possible answer to the question: Venus is unlike Earth because it is significantly closer to the Sun. There is no indication that the author feels regretful, or sorry, about this conclusion.. Choice (B) is incorrect. The statement in lines 43-44 decisively, or without doubt, answers the question posed at the beginning of the paragraph, "Why is Venus so unlike Earth?" The authors response, beginning with the words, "The answer can only lie . . ." indicates that there is only one possible answer to the question: Venus is unlike Earth because it is significantly closer to the Sun. There is nothing guarded, or cautious, about this statement.. Choice (C) is incorrect. The statement in lines 43-44 decisively, or without doubt, answers the question posed at the beginning of the paragraph, "Why is Venus so unlike Earth?" The authors response, beginning with the words, "The answer can only lie . . ." indicates that there is only one possible answer to the question: Venus is unlike Earth because it is significantly closer to the Sun. There is no indication that the author is skeptical, or uncertain, about this conclusion.. Choice (E) is incorrect. The statement in lines 43-44 decisively, or without doubt, answers the question posed at the beginning of the paragraph, "Why is Venus so unlike Earth?" The authors response, beginning with the words, "The answer can only lie . . ." indicates that there is only one possible answer to the question: Venus is unlike Earth because it is significantly closer to the Sun. There is no indication that the author is amused, or entertained, by this conclusion. 16 Explanation for Correct Answer C. Choice (C) is correct. In the sentence beginning in line 2, the narrator tells us that she was used to the sensual curves of Puerto Rico and had to adjust to the regular, aggressive two-dimensionality of Brooklyn. She sets up a contrast between a place of sensual curves, dynamic and pleasing to the senses, and a place she considers regular. In this

context, the term regular is closest in meaning to unvarying, or uniform and changeless; the narrator sees New York as a "hard" place that is flat and two-dimensional, not curved and three-dimensional like Puerto Rico.. Choice (A) is incorrect. In the sentence beginning in line 2, the narrator tells us that she was used to the sensual curves of Puerto Rico and had to adjust to the regular, aggressive two-dimensionality of Brooklyn. She sets up a contrast between a place of sensual curves, dynamic and pleasing to the senses, and a place she considers regular. Although the term regular can describe something that is commonly practiced, in this context it is not closest in meaning to customary: it is somewhat illogical to describe a citys two-dimensional, or flat, appearance as commonly practiced.. Choice (B) is incorrect. In the sentence beginning in line 2, the narrator tells us that she was used to the sensual curves of Puerto Rico and had to adjust to the regular, aggressive two-dimensionality of Brooklyn." She sets up a contrast between a place of sensual curves, dynamic and pleasing to the senses, and a place she considers regular. Something that is agreeable is pleasing to the senses, so the narrator would likely say that Puerto Rico is agreeable and that New Yorkwhich she sees as a dark and dirty place is not agreeable. In this context, the term regular is closest in meaning to unvarying, or uniform.. Choice (D) is incorrect. In the sentence beginning in line 2, the narrator tells us that she was used to the sensual curves of Puerto Rico and had to adjust to the regular, aggressive two-dimensionality of Brooklyn. She sets up a contrast between a place of sensual curves, dynamic and pleasing to the senses, and a place she considers regular. Although the term regular can describe something that occurs at intervals, in this context it is not closest in meaning to recurring: it is somewhat illogical to describe a citys twodimensional, or flat, appearance as recurring.. Choice (E) is incorrect. In the sentence beginning in line 2, the narrator tells us that she was used to the sensual curves of Puerto Rico and had to adjust to the regular, aggressive two-dimensionality of Brooklyn. She sets up a contrast between a place of sensual curves, dynamic and pleasing to the senses, and a place she considers regular. In this context, the term regular is not closest in meaning to average. Something that is average is common or ordinary. The narrator describes having to adjust to her new environment, a place she considers dark and dirty and having an aggressive twodimensionality, or flatness. Although other residents might consider this environment average, it is extraordinary for the narrator, who is used to another environment. 17 Explanation for Correct Answer B. Choice (B) is correct. When the narrator describes how the raindrops look as they fall and hit the ground, she employs vivid imageryfigurative language meant to give the reader a picture of what is going on. The narrator tells us that the raindrops capture the dim silver glow of street lamps, make glistening sparks as they hit the sidewalks, and then

disappear like tiny ephemeral jewels. These vivid descriptions allow the reader to see the scene as the narrator sees it.. Choice (A) is incorrect. Although there is some despair in the first paragraphthe narrator is disappointed and finds New York dark and dirtythe mood of the sentence in lines 48 is not despairing. Instead, the narrator uses vivid imagery (the dim silver glow of street lamps, glistening sparks, tiny ephemeral jewels) to describe raindropsraindrops that, as she reveals later in the paragraph, contain the promise of light and give the author hope.. Choice (C) is incorrect. The narrator uses figurative language in lines 4-8, but she does not use humorous wordplay. Rather, she uses vivid imagery to describe a scene. The narrator tells us that the raindrops capture the dim silver glow of street lamps, make glistening sparks as they hit the sidewalks, and then disappear like tiny ephemeral jewels. These vivid descriptions help the reader to see the raindrops as the narrator sees them; the descriptions are not particularly humorous, or intentionally funny.. Choice (D) is incorrect. Although the narrator has left one home (Puerto Rico) for another (New York), the sentence in lines 4-8 is not nostalgic, or wistfully yearning. In these lines the narrator uses vivid imagery as she describes how the raindrops look in New York, where she now lives. She sees the promise of light in the raindrops as they capture the silver glow of street lamps, make glistening sparks, and disappear like tiny ephemeral jewels. The raindrops make the narrator feel hopeful about her disappointing new home, not nostalgic.. Choice (E) is incorrect. The narrator uses vivid imagery, a type of figurative language, in lines 4-8, but she does not use abstract language. Rather, the narrator uses fairly concrete language when she describes the raindrops as capturing the dim silver glow of street lamps, making glistening sparks as they hit the sidewalks, and disappearing like tiny ephemeral jewels. These vivid descriptions help the reader to see the raindrops as the narrator sees them. 18 Explanation for Correct Answer E. Choice (E) is correct. In lines 8-9, Mami and Tata tease the narrator, saying that she thought the streets of New York would be paved with goldin other words, that she had an unrealistic expectation that New York would be a sort of fantasy land of wealth and prosperity. Mami and Tata think the narrator is disillusioned, or disappointed, because instead of finding the fantasy land she imagined, the narrator finds New York to be dirty and dark.. Choice (A) is incorrect. In lines 8-9, Mami and Tata tease the narrator, saying that she thought the streets of New York would be paved with gold. To say that a places streets are paved with gold is to suggest that it is a fantasy land of wealth and prosperity. The narrator and her family may very well have faced economic barriers, but in these lines,

Mami and Tata are suggesting that the narrator is disillusioned, or disappointed, in New York because she had held unrealistic expectations.. Choice (B) is incorrect. In lines 8-9, Mami and Tata tease the narrator, saying that she thought the streets of New York would be paved with gold. To say that a places streets are paved with gold is to suggest that it is a fantasy land of wealth and prosperity. The narrator might or might not exhibit driving ambitions, but in these lines, Mami and Tata are suggesting that the narrator is disillusioned, or disappointed, in New York because she had held unrealistic expectations.. Choice (C) is incorrect. In lines 8-9, Mami and Tata tease the narrator, saying that she thought the streets of New York would be paved with gold. To say that a places streets are paved with gold is to suggest that it is a fantasy land of wealth and prosperity. The narrator might or might not believe in miraculous transformations, but in these lines, Mami and Tata are suggesting that the narrator is disillusioned, or disappointed, in New York because she had held unrealistic expectations.. Choice (D) is incorrect. In lines 8-9, Mami and Tata tease the narrator, saying that she thought the streets of New York would be paved with gold. To say that a places streets are paved with gold is to suggest that it is a fantasy land of wealth and prosperity. Mami and Tata might or might not think that the narrator is ruled by greedy impulses, but in these lines, they are simply suggest that the narrator is disillusioned, or disappointed, in New York because she had held unrealistic expectations; the narrator expected a fantasy land but instead finds New York to be dirty and dark. 19 Explanation for Correct Answer B. Choice (B) is correct. The narrators description of Brooklyn indicates that she finds it to be uniform, or unvaried and unchanging; she states that everywhere [she looks] she sees a vertical maze of gray and brown . . . buildings. Her description also suggests that she finds Brooklyn to be oppressive, or depressing to the spirit or senses. The buildings have sharp corners and deep shadows, there are cement playground[s] and weedy lots, and there is no horizon; these elements of the environment could certainly create an oppressive feeling or mood.. Choice (A) is incorrect. The narrator briefly expresses some wonder in the second paragraph (wondering where New York ended), but the paragraph as a whole does not indicate that the author finds Brooklyn to be mysterious and unknowable, or beyond human understanding. Rather, the narrators statements suggest that she finds her new environment to be uniform, or unvaried (everywhere I looked my eyes met a vertical maze) and oppressive, or depressing to the spirit or senses (buildings with sharp corners, cement playground, weedy lots . . . and rusting cars).. Choice (C) is incorrect. The second paragraph suggests that the narrator might describe Brooklyn as orderly, or arranged in some order or pattern, because she says that

everywhere [she looks] she sees a vertical maze of gray and brown . . . buildings. But the narrator does not seem to find her new environment appealing, or pleasing: she states that Brooklyn has buildings with sharp corners and deep shadows, cement playground[s] with chain-link fence[s], weedy lots, and no horizon.. Choice (D) is incorrect. The second paragraph suggests that the narrator might describe Brooklyn as drab, or dull and monotonous, because she mentions seeing gray and brown . . . buildings and cement everywhere. But it is unlikely that the narrator finds Brooklyn both drab and multifaceted; something that is multifaceted has many different aspects and is unlikely to be described as monotonous.. Choice (E) is incorrect. The second paragraph suggests that the narrator might find certain aspects of Brooklyn to be menacing, or threatening, because she describes elements of the environment that seem harsh (sharp corners and deep shadows, cement playground, weedy lots . . . and rusting cars), but nothing in the paragraph indicates that she also finds Brooklyn alluring, or charming and enticing. Instead, the narrator seems to find her new environment oppressive, or depressing to the spirit and senses. 20 Explanation for Correct Answer E. Choice (E) is correct. In lines 23-30, the narrator describes her initial interaction with the girl next door. The girl first appraised [the narrator] shyly while the narrator pretended to ignore [the girl]. The narrator then watched as the girl began playing with her jump rope. Before finally speaking, the two girls hesitantly and cautiously took note of, or observed, each other.. Choice (A) is incorrect. In lines 23-30, the narrator describes her initial interaction with the girl next door. Nothing in these lines indicates that the girls were not in the mood to meet someone new. At first the narrator pretended to ignore [the girl] and the girl shyly looked at the narrator, but then the girls smiled at each other and began to talkthe girls actually seemed interested in meeting each other, even though they were cautious at first.. Choice (B) is incorrect. In lines 23-30, the narrator describes her initial interaction with the girl next door. Although at first both girls seem to hide their emotions somewhat, nothing in these lines indicates that either of the girls was afraid. Rather than fearing each other, the girls seem cautiously interested in meeting each other; after a little observation, the girls smiled at each other and began to talk.. Choice (C) is incorrect. In lines 23-30, the narrator describes her initial interaction with the girl next door. The girls did not act as if they already knew each other; instead of greeting one another with any familiarity, the girls observed each other carefully before even smiling.. Choice (D) is incorrect. In lines 23-30, the narrator describes her initial interaction with the girl next door. Although the girls carefully observed each other before speaking, nothing in

these lines indicates that they studied each other with any suspicion, or mistrust. Rather, it seems that the narrator might have pretended to ignore [the girl] out of shyness, and the narrator tells us that the girl appraised [her] shyly. The girls simply took note of each other before smiling and speaking. 21 Explanation for Correct Answer D. Choice (D) is correct. In lines 31-54 the narrator and the girl discuss the label Hispanic. After asking the narrator if she is Hispanic (T eres hispana?), the girl explains that herein New Yorkanybody who speaks Spanish is labeled Hispanic. After the narrator asks for clarification (You mean, if you speak Spanish, youre Hispanic?), the girl attempts to explain that this label has to do with having Puerto Rican or Cuban parents or being from a Spanish country . . . even if you dont speak Spanish. In short, the girls are attempting to identify the criteria that determine the ethnic label.. Choice (A) is incorrect. In lines 31-54 the narrator and the girl discuss the label Hispanic. The girls do talk about speaking Spanish, and about being labeled as Hispanic as a result of speaking Spanish, but the conversation is not a debate over the power of language to shape personality; the girls do not discuss personality at all. Instead, the girls attempt to identify the criteria that determine whether one is considered Hispanic.. Choice (B) is incorrect. In lines 31-54 the narrator and the girl discuss the label Hispanic. Although they are discussing an ethnic label, the girls do not discuss the value of using such labels to characterize people; they simply attempt to identify the criteria that determine when such a label is used.. Choice (C) is incorrect. In lines 31-54 the narrator and the girl discuss the label Hispanic. Although the narrator, who just moved to New York, might find her new environment to be mystifying, the girls do not discuss strategies for survival in a mystifying culture. Instead, the girls attempt to identify the criteria that determine when an ethnic label Hispanicis used.. Choice (E) is incorrect. In lines 31-54 the narrator and the girl discuss the label Hispanic. The girls discuss certain groups (Puerto Rican, Cuban), but they do not attempt to reconcile, or restore harmony between, their group identity and their personal autonomy or freedom. The girls do not discuss personal freedom and how it relates to group identity. Rather, the girls simply attempt to identify the criteria that determine when an ethnic label Hispanicis used. 22 Explanation for Correct Answer B. Choice (B) is correct. The narrators statements in lines 53-55 suggest that for her, being considered Hispanic represents the loss of her former identity as Puerto Rican, which she

has always been. The narrator didnt know that she would be someone else in New York.. Choice (A) is incorrect. The narrators statements in lines 53-55 do suggest that the narrator is experiencing the end of something, but the paragraph does not suggest that she is experiencing the end of her childhood. Instead, the narrator feels that being considered Hispanic represents the loss of her former identity as Puerto Rican, which she has always been.. Choice (C) is incorrect. Although the narrator seems to be somewhat uncomfortable with being labeled Hispanicearlier in the passage she responds, No, Im Puerto Rican when asked if she is Hispanicher statements in lines 53-55 do not suggest that being considered Hispanic is a restriction, or something limiting. Instead, the narrator feels that being considered Hispanic represents the loss of her former identity as Puerto Rican, which she has always been.. Choice (D) is incorrect. The narrator is reacting to being defined in a new waybeing considered Hispanicbut her statements in lines 53-55 do not suggest that she feels that this is an opportunity for self-redefinition. Instead, the narrator feels that this new definition represents the loss of her former identity as Puerto Rican, which she has always been.. Choice (E) is incorrect. The narrator emigrated from Puerto Rico to New York, but her statements in lines 53-55 do not suggest that she feels being considered Hispanic is an unavoidable result of emigration. Indeed, earlier in the passage the author expresses slight confusion at the label; when asked if she is Hispanic, she responds, No, Im Puerto Rican. Instead of feeling that being considered Hispanic is unavoidable, the narrator feels that it represents the loss of her former identity as Puerto Rican, which she has always been. 23 Explanation for Correct Answer E. Choice (E) is correct. In lines 58-61, the narrator mentions talking to a girl, and her mother asks, Where did you meet a girl? Upon finding that the narrator met the girl outside, the mother asks, Who said you could go out to the sidewalk? The mother then states in line 64 that This isnt Puerto Rico and adds (in Spanish) that something could happen to the narrator. When she says This isnt Puerto Rico the mother means that the narrator shouldn't play outside unsupervised like she did in Puerto Rico; in New York, a variety of dangers could be waiting outside the locked doors of the apartment. The mother is trying to impress upon the narrator that different rules apply to life in New York.. Choice (A) is incorrect. In lines 58-61, the narrator mentions talking to a girl, and her mother asks, Where did you meet a girl? Upon finding out that the narrator met the girl outside, the mother asks, Who said you could go out to the sidewalk? The mother then states in line 64 that This isnt Puerto Rico and adds (in Spanish) that something could

happen to the narrator. The narrator and her mother are not discussing nostalgia, or wistful longing, for Puerto Rico in these lines. Instead, when the mother says This isnt Puerto Rico she means that the narrator cant behave as she used toshe cant play outside unsupervisedbecause different rules apply to life in New York.. Choice (B) is incorrect. In lines 58-61, the narrator mentions talking to a girl, and her mother asks, Where did you meet a girl? Upon finding out that the narrator met the girl outside, the mother asks, Who said you could go out to the sidewalk? The mother then states in line 64 that This isnt Puerto Rico and adds (in Spanish) that something could happen to the narrator. The narrator and her mother are not discussing cultural backgrounds or New Yorkers reaction to them in these lines. Instead, when the mother says This isnt Puerto Rico she means that the narrator cant behave as she used toshe cant play outside unsupervisedbecause different rules apply to life in New York. The mother feels that a variety of dangers could be waiting outside the locked doors of the apartment.. Choice (C) is incorrect. In lines 58-61, the narrator mentions talking to a girl, and her mother asks, Where did you meet a girl? Upon finding out that the narrator met the girl outside, the mother asks, Who said you could go out to the sidewalk? The mother then states in line 64 that This isnt Puerto Rico and adds (in Spanish) that something could happen to the narrator. The mother does not refer to Puerto Rico to suggest that newcomers must embrace New York; on the contrary, the mother fears that a variety of dangers could be waiting outside the locked doors of the apartment in New York, so she reminds her daughter that she cannot behave as she did in Puerto Rico.. Choice (D) is incorrect. In lines 58-61, the narrator mentions talking to a girl, and her mother asks, Where did you meet a girl? Upon finding out that the narrator met the girl outside, the mother asks, Who said you could go out to the sidewalk? The mother then states in line 64 that This isnt Puerto Rico and adds (in Spanish) that something could happen to the narrator. The mother is not stating that life was more restricted in Puerto Rico; on the contrary, she suggests that the narrator had more freedom in Puerto Rico. The mother fears that a variety of dangers could be waiting outside the locked doors of the apartment in New York, so she reminds her daughter that she cannot behave as she did in Puerto Rico. 24 Explanation for Correct Answer D. Choice (D) is correct. At the end of the passage, the narrators mood is one of fearfulness and uncertainty. Her sense of identity has been challengedshe has always been Puerto Rican but is now considered Hispanicand she has been reminded that a variety of dangers could lurk outside the apartment doors. In just two days in New York, the narrator has already experienced great change, and she is fearful and uncertain as she imagines that greater dangers lay ahead.. Choice (A) is incorrect. The narrators mood at the end of the passage is not apatheticor lacking concernand sullen, or gloomy or resentfully silent. The narrator does seem

concerned about her future, but rather than being gloomy or resentful, she seems to be fearful and uncertainshe had already experienced great change in just two days in New York, and she quaked as she imagined that greater dangers lay ahead.. Choice (B) is incorrect. The narrators mood at the end of the passage is not best characterized as angry and bewildered, or confused. Rather, the narrator seems fearful and uncertainshe had already experienced great change in just two days in New York, and she quaked as she imagined that greater dangers lay ahead.. Choice (C) is incorrect. The narrators mood at the end of the passage is not best described as one of defeat and resignation, or of being overcome and giving up. Rather, the narrator seems fearful and uncertainshe had already experienced great change in just two days in New York, and she quaked as she imagined that greater dangers lay ahead.. Choice (E) is incorrect. The narrators mood at the end of the passage is not best described as one of resentment and defiance, or hostility and a tendency to resist. Rather, the narrator seems fearful and uncertainshe had already experienced great change in just two days in New York, and she quaked as she imagined that greater dangers lay ahead. Section #9: View Explanations 1 Explanation for Correct Answer C. Choice (C) is correct. Desolate means barren or lifeless. The paintings are described as reflecting the isolation and loneliness of the American Southwest landscape. It makes sense to describe such a landscape as barren or lifeless, so the term desolate logically completes the sentence.. Choice (A) is incorrect. Lush means fertile or thriving, especially with luxuriant foliage. The paintings are described as reflecting the isolation and loneliness of the American Southwest landscape. The term lush does not logically complete the sentence, because a landscape characterized by isolation and loneliness is unlikely to be described as fertile and thriving.. Choice (B) is incorrect. Sprawling means spread out widely and irregularly. The paintings are described as reflecting the isolation and loneliness of the American Southwest landscape. An isolated and lonely landscape is not necessarily spread out widely, so the term sprawling is not the best choice. Another choice, desolate, logically completes the sentence because it describes something barren or lifeless.. Choice (D) is incorrect. Gaudy means outlandish and extravagantly showy. The paintings are described as reflecting the isolation and loneliness of the American Southwest landscape. The term gaudy does not logically complete the sentence, because a landscape

characterized by isolation and loneliness is very unlikely to be described as outlandish and showy.. Choice (E) is incorrect. Monumental means massive. The paintings are described as reflecting the isolation and loneliness of the American Southwest landscape. A landscape might be described as massive, but an isolated and lonely landscape is not necessarily massive, so the term monumental is not the best choice. Another choice, desolate, logically completes the sentence because it describes something barren or lifeless. 2 Explanation for Correct Answer B. Choice (B) is correct. The structure of the sentence indicates that the missing word is defined as a plant produced by . . . crossbreeding two different varieties of flowers. A hybrid is an offspring of two animals or plants of different breeds, varieties, or species, so the term hybrid fits the blank perfectly.. Choice (A) is incorrect. The structure of the sentence indicates that the missing word is defined as a plant produced by . . . crossbreeding two different varieties of flowers. A misnomer is a wrong name or inappropriate designation, not a plant produced from different flowers, so the term misnomer does not logically complete the sentence.. Choice (C) is incorrect. The structure of the sentence indicates that the missing word is defined as a plant produced by . . . crossbreeding two different varieties of flowers. A vector is an organism, such as an insect, that transmits pollen, not a plant produced from different flowers, so the term vector does not logically complete the sentence.. Choice (D) is incorrect. The structure of the sentence indicates that the missing word is defined as a plant produced by . . . crossbreeding two different varieties of flowers. A curative is something that cures a disease. A curative might be plant-based, but it is not necessarily a plant produced from different flowers, so the term curative does not logically complete the sentence.. Choice (E) is incorrect. The structure of the sentence indicates that the missing word is defined as a plant produced by . . . crossbreeding two different varieties of flowers. A precursor is something that precedes something else. The orchid might precede something else, but a precursor is not necessarily a plant produced from different flowers, so the term precursor does not logically complete the sentence. 3 Explanation for Correct Answer A. Choice (A) is correct. Stringent means marked by rigor or strictness. The pharmaceutical companys method of testing is described as more rigorous than the industry standard.

This means that the companys standards were stricter than the industrys norm. Therefore, it makes sense to say that the companys testing was stringent.. Choice (B) is incorrect. Dispersive means spread out or broken up. The sentence mentions the rigor, or strictness, of the pharmaceutical companys drug testing, not whether this testing has spread (for example, to companies across the pharmaceutical industry). Nothing in the sentence suggests that the companys testing was dispersive.. Choice (C) is incorrect. Conditional means dependent upon a premise. A pharmaceutical companys drug testing may be based upon a certain idea or hypothesis, but the sentence mentions only the rigor, or strictness, of this testing, not the theory upon which it was based. Nothing in the sentence suggests that the companys testing was conditional.. Choice (D) is incorrect. Recessive means withdrawn, or detached, and tending to recede. The sentence mentions the pharmaceutical companys drug testing, which was more rigorous, or stricter, than the industrys norm. It is somewhat illogical to suggest that a companys drug testing was withdrawn or detached; additionally, there is no logical connection between being recessive and being strict.. Choice (E) is incorrect. Obtrusive means forward in manner or conduct. The pharmaceutical companys method of testing is described as more rigorous, or stricter, than the industry standard. Although a person who is strict might behave in an obtrusive way, it does not make sense to describe drug tests as forward in conduct. 4 Explanation for Correct Answer D. Choice (D) is correct. Divisive means creating disunity. To restrict something is to limit or prevent it. The structure of the sentence indicates that the term that fits the second blank will contrast in meaning to the term encourage. Restricting, or limiting, something is the opposite of encouraging it, so the term restricted makes sense in the second blank. Further, the structure of the sentence indicates that the part of the sentence after the colon will support the idea presented in the first part of the sentence. The idea that freedom of expression is not always divisive is supported by the fact that encouraging freedom of expression leads to less social unrest, or disunity, in a community.. Choice (A) is incorrect. Revolutionary means bringing about a fundamental change. To promote something is to contribute to its growth. The structure of the sentence indicates that the term that fits the second blank will contrast in meaning to the term encourage. Promoting something is nearly the same as encouraging it, so the term promoted does not create the necessary contrast. Further, the structure of the sentence indicates that the part of the sentence after the colon will support the idea presented in the first part of the sentence; if communities encourage freedom of expression and experience a decrease in social unrest, these communities might consider freedom of expression a revolutionary force..

Choice (B) is incorrect. Positive means having a good or favorable effect. To prohibit something is to forbid it by authority. The structure of the sentence indicates that the term that fits the second blank will contrast in meaning to the term encourage. Prohibiting, or forbidding, something is the opposite of encouraging it, so the term prohibited makes sense in the second blank. But the structure of the sentence also indicates that the part of the sentence after the colon will support the idea presented in the first part of the sentence. A decrease in social unrest is favorable, so the idea that encouraging freedom of expression can lead to such a decrease does not support the idea that freedom of expression is not always a positive force.. Choice (C) is incorrect. Successful means resulting in success or a favorable outcome. To protest something is to make a statement in objection to it. The structure of the sentence indicates that the term that fits the second blank will contrast in meaning with the term encourage. Protesting, or making statements against, something, can be the opposite of encouraging it; however, dissent, or disagreement, can be a form of protest, so it is somewhat illogical to suggest that dissent is protested in some communities. Further, the structure of the sentence indicates that the part of the sentence after the colon will support the idea presented in the first part of the sentence. A decrease in social unrest is favorable, so the idea that encouraging freedom of expression can lead to such a decrease does not support the idea that freedom of expression is not necessarily successful.. Choice (E) is incorrect. In this context, militant means combative and aggressively active. To foster something is to promote its growth or development. The structure of the sentence indicates that the term that fits the second blank will contrast in meaning to the term encourage. Fostering something is nearly the same as encouraging it, so the term fostered does not create the necessary contrast. Further, it is somewhat illogical to describe the concept of freedom of expression as militant, or combative. 5 Explanation for Correct Answer A. Choice (A) is correct. Bucolic means relating to rural life. Because Thomas Hardys novels portray daily life in rural and agricultural settings, it makes sense to use the term bucolic to describe them.. Choice (B) is incorrect. Prolific means characterized by abundant inventiveness or productivity. Although Hardy might be considered a prolific author, it does not make sense to suggest that his novels are prolific, or very productive. Further, the sentence indicates that the novels are described in a certain way because they are about life in rural and agricultural settings, and this subject matter is not related to being prolific.. Choice (C) is incorrect. Lugubrious means mournful, dismal, or gloomy. The sentence indicates that Hardys novels are described in a certain way because they are about life in rural and agricultural settings. Rural and agricultural settings are not necessarily dismal or gloomy, so there is no reason to suggest that one would describe novels about life in such settings as lugubrious..

Choice (D) is incorrect. Sundry means miscellaneous, or consisting of diverse things. The sentence indicates that Hardys novels are preoccupied with, or excessively concerned with, one thingdaily life in rural and agricultural settings. If Hardy was preoccupied with one subject, it is unlikely that his novels would be described as sundry.. Choice (E) is incorrect. Metaphorical means treated as a metaphor. The sentence indicates that the novels are described in a certain way because they are about life in rural and agricultural settings; there is no reason to believe that novels about life in such settings would be described as metaphorical. 6 Explanation for Correct Answer E. Choice (E) is correct. Wrongheaded means contrary to sound judgment. Chicanery is trickery or deception. It is reasonable to assume that some skeptics who think the search for extraterrestrials is foolish would also deem it wrongheaded. The sentence indicates that other skeptics go beyond simply deeming SETI foolish; some even accuse SETI scientists of chicanery, or trickery, claiming that the scientists use skewed, or distorted, data.. Choice (A) is incorrect. Misguided means led or prompted by wrong motives. Remonstrance is an earnest presentation of reasons for opposition. Some skeptics who think the search for extraterrestrials is foolish might also say it is led by wrong motives, so the term misguided makes sense in the first blank. But the term remonstrance does not logically complete the sentence; an earnest presentation is not an offense, so it is unlikely that skeptics would accuse the scientists of making such presentations.. Choice (B) is incorrect. Absurd means ridiculously unreasonable or unsound. Erudition is extensive knowledge acquired chiefly from books. Some skeptics who think the search for extraterrestrials is foolish might also say it is ridiculously unsound, so the term absurd makes sense in the first blank. But the term erudition does not logically complete the sentence. Extensive knowledge acquired from books is not an offense, so it is unlikely that skeptics would accuse the scientists of having such knowledge. Further, there is no reason to believe that people who think the scientists have extensive knowledge would also think that they use skewed, or distorted, data.. Choice (C) is incorrect. Plausible means appearing worthy of belief. Lassitude is weariness or listlessness. Because skeptics are people who doubt things, it is unlikely that skeptics would consider the search for extraterrestrials to be both foolish and plausible; rather, skeptics would say that SETI is foolish and not worthy of belief. Additionally, nothing in the sentence suggests that anyoneskeptical or otherwisewould consider the SETI scientists to be listless or weary.. Choice (D) is incorrect. Painstaking means showing diligent care and effort. In this context, fabrication is the act of lying or producing a falsehood. It makes sense to say

that some skeptics would accuse SETI scientists of lying if they think the scientists are using skewed, or distorted, data, so the term fabrication makes some sense in the second blank. But the term painstaking does not make sense in the first blank. Skeptics who consider the search for extraterrestrials foolish probably would not also characterize the search as showing diligent effort and care. 7 Explanation for Correct Answer C. Choice (C) is correct. The author of Passage 1 clearly states that comic books . . . interfere with education." He or she believes that children gain nothing of educational value from comic books. The author of Passage 2 defends comic books as a "relief zone" for overworked children, but he or she would likely agree with the author of Passage 1 that comic books are not "educational," arguing that comic books "are junk" and that "education is not the purpose of junk.". Choice (A) is incorrect. The author of Passage 1 would agree that comic books impair the social development of children. This is clearly indicated when the author claims that children "do not learn about any normal aspects of sex, love, or life by reading comic books. In contrast, the author of Passage 2 admits that comic books are junk, but goes on to argue that they have a positive value: children can use them as a "relief zone," that is, a respite from pressures placed on them by adults. Therefore, the second author differs from the first in that he or she suggests that comic books may in fact help children in their social development.. Choice (B) is incorrect. Nothing in the passages suggests either author would agree that comic books could benefit from self-regulation. The author of Passage 1 completely dismisses the value of comic books and does not consider how these books could be regulated or changed for the better. The author of Passage 2 expresses an appreciation of comic books antisocial function and does not suggest that they be regulated in any way.. Choice (D) is incorrect. The author of Passage 1 would agree that children have easy access to comic books. The author implies this when he or she states that children spend a large amount of their time and money on these publications. However, the author of Passage 2 would not agree. The phrase "obtained too easily" suggests that there is something harmful about comic books. The author of Passage 2 disagrees with this implication, explaining that comic books help children cope with the pressures placed on them by adults. This author would not say that comic books are obtained too easily.. Choice (E) is incorrect. The author of Passage 1 does not suggest that comic books are garishly amusing. He or she has nothing positive to say about comic books and refuses to even acknowledge that comic books amuse their readers. The author of Passage 2, however, suggests that comic books are both garishthat is, vulgar or offensiveand amusing. To the critics charges that comic books contribute to juvenile delinquency and that they are a corrupting influence, glorifying crime and depravity, the author answers:

But of course. Why else read them? This suggests that, unlike the first author, the second believes in and appreciates the entertainment value of comic books. 8 Explanation for Correct Answer A. Choice (A) is correct. In this context, a matter is a subject or an issue under consideration. In lines 3-4, the author states that "education is not merely a question of learning." The author is not asking a question; the author is using the term question to refer to the matter, or the issue, of learning.. Choice (B) is incorrect. The term request suggests an inquiry. In line 4, the term question is not used to inquire or to ask for something. Rather, it is used to refer to the matter, or the issue, of learning.. Choice (C) is incorrect. The term objection denotes opposition or protest. In line 4, the term question is not used to oppose something. Rather, it is used to refer to the matter, or the issue, of learning.. Choice (D) is incorrect. The term possibility suggests likelihood or chance. In line 4, the term question is not used to suggest the possibility or the likelihood of learning. Rather, it is used to refer to the matter, or the issue, of learning.. Choice (E) is incorrect. The term doubt implies uncertainty or disbelief. In line 4, the author does not use the term question to suggest doubt. Rather, he or she addresses the matter, or the issue, of learning. 9 Explanation for Correct Answer C. Choice (C) is correct. Despite his or her general objection to comic books, the author of Passage 1 does believe that entertainment can have educational value (A great deal of learning comes in the form of entertainment). The author is making a complex point in this part of the passage: he or she is not objecting to comic books because they are a form of entertainment ("a great deal of entertainment painlessly teaches important things"), but because they are a type of entertainment that provides no educational value whatsoever ("They fail to teach anything that might be useful to a child"). The author does not make a sharp distinction between entertainment and learning"; he or she believes that entertainment and learning are closely interrelated.. Choice (A) is incorrect. Even though the author claims that A great deal of learning comes in the form of entertainment, he or she does not necessarily state that educators are entertainers. Rather, the author is making a complex point in this part of the passage: he or she is not objecting to comic books because they are a form of entertainment ("a great deal of entertainment painlessly teaches important things"), but because they are a type of entertainment that provides no educational value whatsoever ("They fail to teach anything

that might be useful to a child"). The author does not make a sharp distinction between entertainment and learning"; he or she believes that entertainment and learning are closely interrelated.. Choice (B) is incorrect. The author of Passage 1 does not imply that one has to be entertained in order to learn; he or she simply states that sometimes learning comes in the form of entertainment. The author is making a complex point in this part of the passage: he or she is not objecting to comic books because they are a form of entertainment ("a great deal of entertainment painlessly teaches important things"), but because they are a type of entertainment that provides no educational value whatsoever ("They fail to teach anything that might be useful to a child"). The author does not make a sharp distinction between entertainment and learning"; he or she believes that entertainment and learning are closely interrelated.. Choice (D) is incorrect. The author of Passage 1 does not believe that comic books can inspire children. According to this author, comic books fail to teach anything that might be useful to a child, and therefore have no redeeming value. The author is making a complex point in this part of the passage: he or she is not objecting to comic books because they are a form of entertainment ("a great deal of entertainment painlessly teaches important things"), but because they are a type of entertainment that provides no educational value whatsoever. The author does not make a sharp distinction between entertainment and learning"; he or she believes that entertainment and learning are closely interrelated.. Choice (E) is incorrect. Nothing in Passage 1 suggests that the author believes that effective textbooks often adopt certain humorous techniques; he or she does not discuss textbooks. The author simply claims that sometimes learning comes in the form of entertainment. The author is making a complex point in this part of the passage: he or she is not objecting to comic books because they are a form of entertainment ("a great deal of entertainment painlessly teaches important things"), but because they are a type of entertainment with no educational value whatsoever ("They fail to teach anything that might be useful to a child"). The author does not make a sharp distinction between entertainment and learning"; he or she believes that entertainment and learning are closely interrelated. 10 Explanation for Correct Answer D. Choice (D) is correct. The three sentences in lines 18-22 highlight the various ways in which childrens education is shortchanged, or deprived, when children spend most of their time reading comic books. The repetition of the word They emphasizes the authors message: children gain nothing of educational valueor any value, for that matterby reading comic books.. Choice (A) is incorrect. The three sentences in lines 18-22 do not lament, or express regret about, students lack of interest in traditional education. Rather, they highlight the ways in

which children are shortchanged, or deprived, when they spend most of their time reading comic books. According to the author, children learn nothing valuable from comic books.. Choice (B) is incorrect. The three sentences in lines 18-22 do not condemn those who produce comic books or those who profit by making these publications available to children. The three sentences beginning with They refer to different ways in which children are shortchanged, or deprived, when they spend most of their time reading comic books. According to the author, children learn nothing valuable from comic books.. Choice (C) is incorrect. The three sentences in lines 18-22 do not enumerate the failings of the educational system. Rather, the sentences primarily express the various ways in which childrens education is shortchanged, or deprived, when children spend most of their time reading comic books. According to the author, it is not the educational system that fails children; it is comic books.. Choice (E) is incorrect. The three sentences in lines 18-22 do not suggest ways to improve comic books. Rather, the sentences outline the various ways in which childrens education is shortchanged, or deprived, when children spend most of their time reading comic books. The repetition of the word They emphasizes the authors message: children gain nothing of educational valueor any value, for that matterby reading comic books. 11 Explanation for Correct Answer E. Choice (E) is correct. In response to the claim in Passage 1 that those who read comic books as children do not save or collect them, the author of Passage 2 would likely contend that there are old comic book fans, a small army of them, who not only save old comic books, but also buy them and trade them. According to the evidence provided in Passage 2, the author of Passage 1 has a limited perspective and may just be wrong: the author of Passage 1 may claim not to know any adult comic book fans, but there are in fact many of them.. Choice (A) is incorrect. The author of Passage 2 would be unlikely to comment on adolescents tendency to be passionate about their likes and dislikes. The author of Passage 2 rebuts the previous authors argument by asserting that people actually do save, trade, and buy comic books long after their childhood and adolescence: there are old comic book fans, a small army of them.. Choice (B) is incorrect. The author of Passage 2 does not support the idea that comic books do not provide lifelong entertainment. In fact, the first paragraph of Passage 2 asserts just the opposite: some people who were comic book fans as children continue to be addicts of these types of books well into their adulthood.. Choice (C) is incorrect. The author of Passage 2 would not likely claim that collectible pop-culture items such as comic books are now in museums. Instead, the author rebuts the previous authors argument by asserting that people actually do save, trade, and buy comic

books long after their childhood and adolescence. While comic books may be saved, traded, and bought by adult fans, there is no indication that comic books are displayed or celebrated at museums.. Choice (D) is incorrect. The author of Passage 2 would not assert that one cannot explain the sentimental value of comic books. In fact, certain parts of Passage 2 do explain their sentimental value. When the author of Passage 2 states that With [comic books] we were able to roam free, disguised in costume, committing the greatest of feats, he or she offers some rationalizations for the sentimental value of comic booksbecause adults associate these enjoyable childhood experiences with reading comic books, they continue to be addicts of these types of books. 12 Explanation for Correct Answer C. Choice (C) is correct. In lines 27-29 the author of Passage 1 makes the claim that "children spend a large amount of their time and money" on comic books but "have nothing positive to show for it." The author of Passage 2 best refutes this claim by showing how children benefit from reading comic books. Beginning in the fourth paragraph of Passage 2, the author explains the more positive side of comic books: they offer children a relief zone from their stressful daily lives.. Choice (A) is incorrect. In lines 27-29 the author of Passage 1 makes the claim that "children spend a large amount of their time and money" on comic books but "have nothing positive to show for it." The author of Passage 2 does not make the argument that comic books do not cost as much as other means of amusement. Neither of the authors is primarily concerned with the cost of entertainment; they are primarily concerned with the educational or social value of comic books.. Choice (B) is incorrect. In lines 27-29 the author of Passage 1 makes the claim that "children spend a large amount of their time and money" on comic books but "have nothing positive to show for it." The author of Passage 2 best refutes this claim by showing how children benefit from reading comic books. He or she explains the more positive side of comic books: they offer children a relief zone from their stressful daily lives. The argument that comic books openly acknowledge their true purpose, while important in some respects, is not especially relevant to the claim made in Passage 1 and is not discussed in either passage.. Choice (D) is incorrect. In lines 27-29 the author of Passage 1 makes the claim that "children spend a large amount of their time and money" on comic books but "have nothing positive to show for it." The author of Passage 2 best refutes this claim by showing how children benefit from reading comic books. He or she explains the more positive side of comic books: they offer children a relief zone from their stressful daily lives. The author of Passage 2 does not discuss comic book readers who might lack a sense of humor..

Choice (E) is incorrect. In lines 27-29 the author of Passage 1 makes the claim that "children spend a large amount of their time and money" on comic books but "have nothing positive to show for it." The author of Passage 2 best refutes this claim by showing how children benefit from reading comic books. He or she explains the more positive side of comic books: they offer children a relief zone from their stressful daily lives. The author of Passage 2 does not make the argument that comic books have never been proven to distract children from homework. 13 Explanation for Correct Answer B. Choice (B) is correct. The quotation marks serve to highlight the term fanzines, an informal, specialized term that refers to nonprofessional publications produced by comic book fans. This term may be unfamiliar to many mainstream readers. As the term implies, a "fanzine" is a magazine created by a fan.. Choice (A) is incorrect. The quotation marks around the term "fanzines" do not serve to underscore the terms traditional definition. The term fanzine is not traditional; it is an informal, specialized term that refers to nonprofessional publications that comic book fans produce in order to glorify an old era of comic books. As the term implies, a "fanzine" is a magazine created by a fan.. Choice (C) is incorrect. The quotation marks around the term "fanzines" do not serve to attribute a novel, or new, concept. While the concept of fanzines, or magazines created by comic book fans, was rather new in 1965 (the year in which this passage was published), the purpose of these quotation marks is not to "attribute" anything. In this context, the word "attribute" means to ascribe, or give credit, to a particular author or source. The quotation marks are used instead to highlight an informal, specialized term that may be unfamiliar to mainstream readers.. Choice (D) is incorrect. The quotation marks around the term "fanzines" do not serve to mock, or make fun of, a flawed hypothesis. The author does not propose a hypothesis in the first paragraph. He or she merely claims that fanzinesspecialized nonprofessional publications produced by comic book fanshelp keep the comic book tradition alive.. Choice (E) is incorrect. The quotation marks around the term "fanzines" do not aim to support a challenging assertion, or claim. The notion that fanzinesspecialized nonprofessional publications produced by comic book fansexist is not a challenging assertion; it is a mere fact. The quotation marks serve to draw attention to the term fanzines, which may be unfamiliar to many mainstream readers. 14 Explanation for Correct Answer D. Choice (D) is correct. In this sentence, the author of Passage 2 is making the point that comic books are intended to be "junk"; that is, they are basically intended to be corrupt,

immoral, and non-educational, so it does not make sense to attack them on these grounds, to "accuse them of being what they are." In short, comic books do not claim to be educationalthere is no such thing as . . . educational junkso why should people complain about them not being educational? The author then adds, as an aside, that "attempts at the latter [educational junk] have, from time to time, been foisted upon us." The author seems to be referring to junk products, including comic books, that have mistakenly tried to be educational. According to his or her argument, these products would necessarily be ill-conceived failures, because education is not the purpose of junk.". Choice (A) is incorrect. In this sentence, the author of Passage 2 is making the point that comic books are intended to be "junk"; that is, they are basically intended to be corrupt, immoral, and non-educational, so it makes no sense to attack them on these grounds, to "accuse them of being what they are." In short, comic books do not claim to be educational there is no such thing as . . . educational junkso why should people complain about them not being educational? The author then adds, as an aside, that "attempts at the latter [educational junk] have, from time to time, been foisted upon us." The author seems to be referring to junk products, including comic books, that have mistakenly tried to be educational. These products would best be described as ill-conceived failures, not as unpolished, or imperfect.. Choice (B) is incorrect. In this sentence, the author of Passage 2 is making the point that comic books are intended to be "junk"; that is, they are basically intended to be corrupt, immoral, and non-educational, so it makes no sense to attack them on these grounds, to "accuse them of being what they are." In short, comic books do not claim to be educational there is no such thing as . . . educational junkso why should people complain about them not being educational? The author then adds, as an aside, that "attempts at the latter [educational junk] have, from time to time, been foisted upon us." The author seems to be referring to junk products, including comic books, that have mistakenly tried to be educational. These products would likely be unpopular (as the author says, "half-hearted attempts to bring reality or literature to comic books invariably look embarrassing"), but they would be better described as ill-conceived failures than as unpopular changes.. Choice (C) incorrect. In this sentence, the author of Passage 2 is making the point that comic books are intended to be "junk"; that is, they are basically intended to be corrupt, immoral, and non-educational, so it makes no sense to attack them on these grounds, to "accuse them of being what they are." In short, comic books do not claim to be educational there is no such thing as . . . educational junkso why should people complain about them not being educational? The author then adds, as an aside, that "attempts at the latter [educational junk] have, from time to time, been foisted upon us." The author seems to be referring to junk products, including comic books, that have mistakenly tried to be educational. These products would best be described as ill-conceived failures, not as misunderstood creations.. Choice (E) is incorrect. In this sentence, the author of Passage 2 is making the point that comic books are intended to be "junk"; that is, they are basically intended to be corrupt, immoral, and non-educational, so it makes no sense to attack them on these grounds, to

"accuse them of being what they are." In short, comic books do not claim to be educational there is no such thing as . . . educational junkso why should people complain about them not being educational? The author then adds, as an aside, that "attempts at the latter [educational junk] have, from time to time, been foisted upon us." The author seems to be referring to junk products, including comic books, that have mistakenly tried to be educational. These products would not be accurately described as imitations; rather, they are misguided attempts to give junk an educational purpose. 15 Explanation for Correct Answer E. Choice (E) is correct. When something is compromised, its value is diminished or degraded. The author of Passage 2 states in line 49 that comic books . . . are junk. Although the author acknowledges that junk has a positive side, he or she explains that these books "entertain on the . . . most compromised"most diminished or degraded"of levels.". Choice (A) is incorrect. In this context, the word compromised does not indicate a settlement or a resolution. Rather, the word signifies a lowering or loss of value. The author of Passage 2 refers to comic books as junk, a compromised or degraded means of expression. Of course, junk has a "positive side" as well, which the author discusses at length.. Choice (B) is incorrect. The word endangered means that something is in danger; it has been imperiled or jeopardized. When the author of Passage 2 uses the word compromised, he or she intends to describe the low, degraded level of entertainment that comic books provide. The author does not imply that comic books are endangered in any way.. Choice (C) incorrect. To combine is to join or unite. While the word compromise can mean to reach consensus by combining or unifying ideas, the word compromisedas it is used in line 57refers to the low, degraded level of entertainment that comic books provide. Of course, comic books have a "positive side" as well, which the author of Passage 2 discusses at length.. Choice (D) is incorrect. To reconcile is to settle or resolve something. While a compromise can be part of the reconciliation process, the word compromisedas it is used in line 57 refers to the low, degraded level of entertainment that comic books provide. Of course, comic books have a "positive side" as well, which the author of Passage 2 discusses at length. 16 Explanation for Correct Answer D. Choice (D) is correct. The term "therapeutic" means providing a cure or treatment for a disease or disorder. The author of Passage 2 argues that it is beneficial for children to enter

the fantasy world of comic books because "children, simply to stay sane, must . . . . have a place to hide where they cannot be got at by grownups. The author states that the basic sustenance for this relief is comic books. In the last sentence of the passage, the author refers to comic book readers as "Psychically renewed" and able to return to their lives of pressure and responsibility. The author is focused on the role comics can play in preserving children's mental health; comic books have a therapeutic, curative effect on young readers.. Choice (A) is incorrect. Rather than tapping into childrens repressed fearsfears that are not expressed or acknowledgedthe fantasy world of comic books offers children an escape from daily pressures. The author of Passage 2 describes this relief zone as a world in which children engage in activities that they would normally not be allowed to engage in, such as roam[ing] free, disguised in costume, committing the greatest of featsand the worst of sins. This description does not imply that those actions reveal childrens repressed fears. In fact, comics are described as providing a fantasy world that empowers children.. Choice (B) is incorrect. Nothing in lines 68-87 indicate that the fantasy world of comic books, which the author of Passage 2 describes as a relief zone for children, fails to stand up to or even warrants extended analysis or scrutiny. On the contrary, it is a world in which children are free from scrutiny: a place to hide where they cannot be got at by grownups.. Choice (C) is incorrect. The fantasy world of comic books may certainly appeal to some adults. However, the author of Passage 2 describes comic books as a relief zone inhabited exclusively by children who need to escape from a strict regimen of work and study that adults and society force upon them. The author is primarily concerned with the effects that comic books have on children, not adults.. Choice (E) is incorrect. Although the first paragraph of Passage 2 suggests that many comic book fans grow up to become successful writers, nowhere does the author indicate that children learn how to write well by reading comic books. Rather, the author describes comic books as a relief zone inhabited exclusively by children who need to escape from a strict regimen of work and study that adults and society force upon them. 17 Explanation for Correct Answer E. Choice (E) is correct. The author of Passage 1 expresses the belief that comic books have no educational value and "suggest many things that are harmful" to children. He or she would most likely regard lines 81-83particularly the description of comic book characters committing the worst of sins" and "getting away with them"as evidence that supports his or her belief in the harmful influence of comic books.. Choice (A) is incorrect. The main argument that the author of Passage 1 makes is that comic books do not have any educational valuein fact, the author states that By no stretch of critical standards can the text in comics qualify as literature. Therefore, he or she would not regard lines 81-83 of Passage 2 as evidence of students inability to read

difficult and challenging literature. Rather, he or she would find proof for his or her argument that comic books "suggest many things that are harmful" to children.. Choice (B) is incorrect. The author of Passage 2 does not indicate that students are secretly reading their comic books at school. Therefore, it is unlikely that the author of Passage 1 would consider the activities described in lines 81-83 to be a result of schools failure to monitor their students. Passage 1 does not suggest in any way that schools are to blame for the fact that some students are reading comic books.. Choice (C) is incorrect. The author of Passage 1 expresses the opinion that A great deal of learning comes in the form of entertainment and that entertainment . . . teaches important things. However, he or she also asserts that children do not gain anything valuable by reading comic books. Therefore, the author of Passage 1 would not regard the description in lines 81-83 of the value of comic books as proof of why we need to combine education with entertainment.. Choice (D) is incorrect. In lines 81-83 the author of Passage 2 describes the wonderful sense of freedom and power that children experience when they read comic books. It is unlikely that even the author of Passage 1 would contend that these profound feelings are a result of "hackneyed"that is, common or overusednarratives in comics. 18 Explanation for Correct Answer C. Choice (C) is correct. The tone of Passage 1 is more "severe," or harsh, than the tone of Passage 2. The author of the first passage expresses a severe tone by repeating, in absolute terms, that comics have no redeeming qualities whatsoever (comics . . . are not only noneducational; they are anti-educational. They fail to teach anything that might be useful to a child; they do suggest many things that are harmful). In contrast, the author of Passage 2 admits that comic books are junk, but goes on to explore ways in which they can be beneficial to children.. Choice (A) is incorrect. While there are conversational features in Passage 1, such as the authors references to his or her own life experiences, the predominant tone in the first passage is "severe," or harsh. Repeatedly insisting that comics have no redeeming value whatsoever (comics . . . are not only non-educational; they are anti-educational. They fail to teach anything that might be useful to a child; they do suggest many things that are harmful), the author of Passage 1 comes across as much more severe than the author of Passage 2.. Choice (B) is incorrect. The tone of Passage 1 is not more "facetious"meant to be humorous or funnythan the tone of Passage 2. Rather, the author of Passage 1 comes across as very serious about the lack of educational and entertainment value of comic books..

Choice (D) is incorrect. The author of Passage 1 is not being sarcastic: he or she does not use humor or irony to ridicule anyone or anything. Rather, the author of Passage 1 seems quite earnest and serious as he or she repeats, in absolute terms, that comics have no redeeming qualities whatsoever (comics . . . are not only non-educational; they are antieducational. They fail to teach anything that might be useful to a child; they do suggest many things that are harmful).. Choice (E) is incorrect. Both passages are somewhat analytical in that they state a point of view, proceed to support it with reasoning and personal observations, and rebut potentially opposing points of view to further emphasize their main idea. But neither passage is exceptionally analytical: one of the main pieces of evidence in Passage 1 is the fact that the author does not know any adult comic book fans, and Passage 2 is quite poetic in describing the effects of comics. It is impossible to say that Passage 1 is more analytical than Passage 2. Section #10: View Explanations 1 Explanation for Correct Answer B. Choice (B) is correct. It avoids the unidiomatic phrasing of the original by providing a verb phrase (would know) that indicates a completed action in the future.. Choice (A) involves an unidiomatic phrase. The verb phrase would be knowing is not idiomatic. The verb to know is not idiomatically described as a continuing action, only a completed action.. Choice (C) results in inappropriate verb tense. The past-tense verb practiced implies that the action of knowing an art should be described as a completed action in the future (would know) instead of an action to occur in the future (will know).. Choice (D) results in illogical phrasing. It does not make sense to say that the children were known to an art.. Choice (E) results in an unidiomatic phrase. The verb phrase will be knowing is not idiomatic. The verb to know is not idiomatically described as a continuous action, only a completed action. 2 Explanation for Correct Answer C. Choice (C) is correct. It avoids the confusing phrasing and wordiness of the original by placing the actions (born, raised, and resides) in chronological order..

Choice (A) involves wordiness and confusing phrasing. The sentence lists the actions (born, raised, and resides) in the reverse order of which they occurred, creating a confusing statement.. Choice (B) results in an inappropriate verb tense. The introductory phrase Being raised in Chile implies continuing action; however, the context of the sentence indicates that this is an action completed in the past.. Choice (D) results in a comma splice. Two independent clauses (Although now in California, Isabel Allende was born in Peru and raised in Chile and she is the author of The House of the Spirits) are joined by only a comma.. Choice (E) results in a lack of parallelism. The verb phrase now she resides in California is not parallel with the preceding modifying phrase raised in Chile. 3 Explanation for Correct Answer C. Choice (C) is correct. It avoids the sentence-fragment error of the original by providing a main verb phrase (was not established) to carry out the action of the sentence.. Choice (A) involves a sentence fragment. There is no main verb to carry out the action of the main clause, only the participial phrase not being established . . .. Choice (B) results in a sentence fragment. There is no main verb to carry out the action of the main clause, only the modifying phrase which had not been established . . .. Choice (D) involves a vague pronoun, resulting in an illogical sentence. The pronoun it can only refer to the voting of women in the Wyoming territory as early as 1869. However, it is not logical to say this voting in the Wyoming territory established suffrage for women throughout the United States upon the ratification of the Nineteenth Amendment in 1920.. Choice (E) results in an inappropriate verb tense. The establishment of suffrage for women is described as a single moment in the past. The verb phrase was not being established inappropriately implies an action taking place over a period of time. 4 Explanation for Correct Answer B. Choice (B) is correct. Two independent clauses (Bees must leave the safety of the hive to forage for food many times a day and they are risking being eaten by any of a multitude of predators) are joined by only a comma.. Choice (A) involves a comma splice. It avoids the comma-splice error of the original by replacing an independent clause with the modifying phrase at the risk of being eaten . . ..

Choice (C) results in awkward, unidiomatic phrasing. It does not make sense to say risking them to be eaten.. Choice (D) results in confusing, illogical phrasing. It does not make sense to say the risk is to be eaten. This illogically implies that the bees intend to be eaten.. Choice (E) results in improper subordination. The semicolon and conjunction likewise do not indicate that the second clause (they risk being eaten . . .) is a result of the first clause (Bees must leave the safety of the hive . . .). 5 Explanation for Correct Answer E. Choice (E) is correct. It avoids the errors of the original by providing an infinitive (to acquire) to indicate what it took the museum half a century to do and a possessive pronoun (its) to indicate that the Picasso collection belongs to the museum.. Choice (A) involves improper coordination. The relationship between the ideas in the two independent clauses (It took . . . effort and it acquired . . . collection) is awkwardly stated and unclear. An infinitive (to acquire) and a possessive pronoun (its) are needed in place of and it acquired and the to indicate what it took the museum half a century to do and that the Picasso collection belongs to the museum.. Choice (B) involves an inappropriate pronoun and results in faulty sentence structure. There is nothing in the sentence to which the plural possessive pronoun their can logically refer. In addition, there is no main verb to carry out the action of the clause before their . . ., only the participle being.. Choice (C) involves improper coordination, awkward, wordy phrasing, and an inappropriate pronoun. The relationship between the ideas in the two independent clauses (It took . . . effort and finally they had . . . there) is awkwardly stated and unclear; an infinitive (to acquire) is needed in place of and finally to indicate what it took the museum half a century to do; and then the awkward, wordy they had an outstanding Picasso collection there can be shortened to its outstanding Picasso collection. Finally, there is nothing in the sentence to which the plural pronoun they can logically refer. Choice (D) involves improper coordination and redundancy. The relationship between the ideas in the two independent clauses (It took . . . effort and but finally an outstanding . . . acquired at last) is awkwardly stated and unclear; an infinitive (to acquire) is needed in order to indicate what it took the museum half a century to do. In addition, finally or at last are redundant; at most one of these phrases should be used. 6 Explanation for Correct Answer E.

Choice (E) is correct. It avoids the modification error of the original by providing a subject (Maria Elena) that can be logically modified by the phrase whose art explores Mexican cultural themes.. Choice (A) involves illogical modification. It does not make sense to say that the art of Maria Elena is An artist who explores Mexican cultural themes.. Choice (B) results in awkward, confusing phrasing. This sentence illogically implies that the work of Maria Elena is world-renowned art in order to explore Mexican cultural themes.. Choice (C) is awkward and alters the meaning of the original sentence. Instead of saying that Maria Elenas art is world renowned, this sentence awkwardly says that the art of Mexican cultural themes is world renowned.. Choice (D) involves illogical modification. It does not make sense to say that Maria Elenas art is An artist who has explored Mexican cultural themes. 7 Explanation for Correct Answer A. Choice (A) is correct. It avoids the errors of the other options by providing verbs in appropriate tenses.. Choice (B) results in a verb-tense error. The sentence describes an attitude in the seventeenth century, so the verb would, which can indicate action in the past, is more appropriate than the future-tense verb will.. Choice (C) results in a verb-tense error. The sentence describes an attitude in the seventeenth century, so the present-tense verb phrase are to teach should be replaced with verb phrase should teach, which can indicate action in the past.. Choice (D) results in a verb-tense error and wordiness. The sentence describes an attitude in the seventeenth century, so the future-tense verb will should be replaced with the verb would, which can indicate action in the past. In addition, the wordy phrase should only teach that which [would] not can be reduced to should teach nothing that would.. Choice (E) results in verb-tense errors. The sentence describes an attitude in the seventeenth century, so the future-tense verbs shall and will are inappropriate and should be replaced with the verbs should and would, which can indicate action in the past. 8 Explanation for Correct Answer D.

Choice (D) is correct. It avoids the lack of parallelism of the original by providing the phrase in foreign languages, which is parallel to the preceding phrase (in math).. Choice (A) involves a lack of parallelism. The phrases joined by and must be parallel. The phrase to learn foreign languages is not parallel with the preceding phrase (in math).. Choice (B) results in a lack of parallelism and vague pronoun reference. The phrases joined by and must be parallel. The phrase that she could do it well in foreign languages is not parallel with the preceding phrase (in math). Furthermore, there is nothing in the sentence to which the pronoun it can logically refer.. Choice (C) results in a lack of parallelism. The phrases joined by as well as must be parallel. The phrase in learning foreign languages is not parallel with the preceding phrase (in math).. Choice (E) results in a lack of parallelism. The phrases joined by and must be parallel. The phrase in foreign languages is not parallel with the preceding phrase (to learn math). 9 Explanation for Correct Answer A. Choice (A) is correct. It avoids the errors of the other options by providing a concise, complete sentence that appropriately uses the both . . . and . . . construction.. Choice (B) results in a sentence fragment. There is no main verb to carry out the action of the sentence, only the modifying phrase which reveal to us . . .. Choice (C) results in awkward, unidiomatic phrasing. There are so many intervening elements between the two parts of the phrase the pain . . . of his boyhood that it is hard to comprehend. In addition, the negative phrase Not only requires inversion of the subject and verb (Not only is the pain of his boyhood . . .).. Choice (D) results in confusing, unidiomatic phrasing. It does not make sense to say that Thomas Wolfe reveals to us both the pain with the beauty of his boyhood in the American South. The correct construction is both . . . and . . ., not both . . . with . . .. Choice (E) results in a sentence fragment. There is no main verb to carry out the action of the sentence, only the participial phrase Thomas Wolfe thus revealing . . . 10 Explanation for Correct Answer B. Choice (B) is correct. It avoids the confusing, illogical phrasing and wordiness of the original by replacing the phrase require for one to cook without there having to be

interruptions with the logical, concise phrase require that one cook without interruption.. Choice (A) involves wordiness and illogical phrasing. The phrase require for one to cook without there having to be interruptions is wordy and illogical (it implies that sometimes following a recipe is possible only if there are interruptions). It can be reduced to the concise, logical require that one cook without interruption.. Choice (C) results in inappropriate phrasing. The introductory phrase Being that is awkward and should be replaced with the more direct Because. In addition, the secondperson pronoun you is inappropriate in this context and should be replaced with one. Finally, the awkward and not be interrupted should be replaced with the precise without interruption.. Choice (D) results in improper subordination. Joining the two clauses (Many Szechuan recipes require that one cook without interruption and it is a good idea to measure all ingredients in advance) with and does not properly indicate that the second clause is a result of the first clause.. Choice (E) results in awkward phrasing. The adverbial phrase When following many Szechuan recipes can be read to mean one is following many recipes at once. 11 Explanation for Correct Answer D. Choice (D) is correct. It avoids the error of the original by comparing two like things (the music of Gershwin and that of most of his contemporaries).. Choice (A) involves an illogical comparison. It does not make sense to compare the music of Gershwin with most of his contemporaries.. Choice (B) results in an illogical comparison. This sentence compares the music of Gershwin with most of his contemporaries.. Choice (C) results in an illogical comparison. It does not make sense to compare the music of Gershwin with most of his contemporaries.. Choice (E) results in awkward, unclear phrasing. It is not clear what the music of Gershwin is being compared with. The phrase as far as music is concerned should be deleted and that of should be inserted after than to create a logical, concise comparison. 12 Explanation for Correct Answer E.

Choice (E) is correct. It avoids the modification error of the original by providing a modifying phrase (thus inaugurating) that logically applies to Allen Ginsberg.. Choice (A) involves illogical modification. It does not make sense to say that Allen Ginsberg was being the inauguration of both a new style of poetry and the Beat movement.. Choice (B) results in vague phrasing and a comma splice. There are no two things in the beginning of the sentence to which both can refer. In addition, two independent clauses (On October 13, 1955, at the Six Gallery in San Francisco, Allen Ginsberg read his poem Howl and both inaugurated a new style in poetry and the Beat movement) are joined by only a comma.. Choice (C) results in a comma splice. Two independent clauses (On October 13, 1955, at the Six Gallery in San Francisco, Allen Ginsberg read his poem Howl and it was the inauguration of both a new style in poetry and the Beat movement) are joined by only a comma.. Choice (D) results in faulty sentence structure. In this context, whose is an adjective that must be followed by a full clause; however, there is no main verb to carry out the action of the whose-clause. 13 Explanation for Correct Answer A. Choice (A) is correct. It avoids the errors of the other options by providing a subject (Canadians) that can be logically modified by the sentences opening phrase, Indicating their desire to extend free enterprise.. Choice (B) results in illogical modification. It does not make sense to say that an election was Indicating their desire to extend free enterprise.. Choice (C) results in illogical modification. It does not make sense to say that Kim Campbell was Indicating their desire to extend free enterprise.. Choice (D) results in illogical modification. It does not make sense to say that Kim Campbell was Indicating their desire to extend free enterprise.. Choice (E) results in illogical modification. It does not make sense to say that a member was Indicating their desire to extend free enterprise. 14 Explanation for Correct Answer C.

Choice (C) is correct. It avoids the awkward, wordy phrasing of the original by replacing the passive phrase it is rare for kindness to be shown with the active phrase they rarely show kindness, which parallels the active construction in the first independent clause.. Choice (A) involves wordiness and awkward, confusing passive voice. The phrase it is rare for kindness to be shown is wordy, and the passive voice clashes with the active voice in the first independent clause and does not indicate who rarely shows kindness to contrary theories.. Choice (B) results in a sentence fragment. There is no main verb, only the participle being.. Choice (D) results in a comma splice. Two independent clauses (Researchers tend to praise studies that agree with their own conclusions and they are rarely kind to contrary theories) are joined by only a comma.. Choice (E) results in faulty sentence structure. The conjunction although calls for an independent clause to follow; instead, there is only the participial phrase rarely showing kindness to contrary theories.

You might also like